You are on page 1of 43

PRAYAS - 2022

Test – 09
Geography
Topics covered:
 India — Location, Structure and Physiography
 Drainage System
 Climate
 Natural Vegetation, Soils
 Mineral resources, Energy resources, Distribution of Industries
Transportation

e
in
nl
l.o

PRAYAS-2022
ria
e
at
cm
ps
.u
w
w
w
w
w
w
.u
ps
cm
at
er
ia
l.o
nl
in
e
Contact us : info@onlyias.com

OnlyIAS Nothing Else Visit : dpp.onlyias.in


Contact : +91-7007 931 912

Q.1) Consider the following statements: Q.5) Consider the following peaks:
1. Mizoram and Nagaland share a border with 1. Saramati
Myanmar only. 2. Nokrek
2. Sikkim is the only state forming an international 3. Makalu
boundary on the three sides. 4. Annapurna
3. Tripura forms the longest boundary with 5. Kamet
Bangladesh. 6. Nanda Devi
Select the incorrect Ans using the codes given below: Which of the following is the correct sequence of the
a) 3 only above peaks in the east to west direction?
b) 2 and 3 only a) 2-1-4-3-5-6
c) 2 only b) 1-2-3-4-6-5
d) 1, 2, and 3 only c) 2-1-4-3-6-5
d) 1-2-4-3-6-5
Q.2) Consider the following statements:
1. The latitude of Jaipur is southwards to that of Q.6) Consider the following statements:
Lucknow. 1. Aravallis are spread across the four Indian states
2. The longitude of Hyderabad lies between Kurnool and UTs.
and Amravati. 2. Himalayas do not extend to the state of Tripura.
3. The latitude of Palghar is northwards to that of 3. The tree line of the western Himalayas is less than
Visakhapatnam. the Eastern.
Select the correct Ans using the codes given below: Select the correct Ans using the codes given below:
a) 1 and 2 only a) 1 and 3 only
b) 2 and 3 only b) 1 and 2 only
c) 1 and 3 only c) 2 and 3 only
d) 1, 2 and 3 d) 1, 2 and 3

Q.3) Consider the following pairs: Q.7) Consider the following statements:
1. Indira col: Andaman and Nicobar Island 1. Karewas are confined to Kashmir while Duns are
2. Kibithu: Nagaland confined to Uttarakhand.
3. Guhar Moti: Gujarat 2. Bugyals are the grasslands located in the valleys of
Which of the above pairs are correctly matched? Uttarakhand.
a) 1 only Select the correct Ans using the codes given below:
b) 1 and 2 only a) 1 only
c) 1 and 3 only b) 2 only
d) 3 only c) Both 1 and 2
d) Neither 1 nor 2
Q.4) Consider the following statements:
1. The Himalayas in the North East India are Q.8) Consider the following statements:
continuously aligned in the North to South 1. Peninsular rivers have greater drainage basins due
e

direction. to their fixed course.


in

2. Meghalaya and the Karbi Anglong hills are the 2. Himalayan rivers exhibit a single drainage pattern
nl

extension of Deccan Plateau. throughout their course.


l.o

3. Duar formations in the Sikkim region is an Select the correct Ans using the codes given below:
extension of the Shivalik. a) 1 only
ria

Select the correct Ans from the codes given below: b) 2 only
e

a) 1 and 3 only c) Both 1 and 2


at

b) 2 and 3 only d) Neither 1 nor 2


cm

c) 1, 2 and 3
d) None of the above Q.9) Consider the following events:
ps
.u

PRAYAS TEST 9 1
w
w
w
Contact us : info@onlyias.com

OnlyIAS Nothing Else Visit : dpp.onlyias.in


Contact : +91-7007 931 912

1. Development of Malda gap 1. The origin of Brahmaputra lies in the Angsi


2. Upliftment of Potwar Plateau Glaciers.
3. Upliftment of western Himalayas 2. It houses the only Island district of India.
4. Emergent Western coasts Select the correct Ans using the codes given below:
Which of the above events are responsible for the a) 1 only
evolution of present-day Himalayan rivers? b) 2 only
a) 1, 2 and 4 only c) Both 1 and 2
b) 1, 2 and 3 only d) Neither 1 nor 2
c) 2, 3, and 4 only
d) 1, 3, and 4 only Q.14) Which of the following National Parks are in
the vicinity of river Brahmaputra?
Q.10) Consider the following statements regarding 1. Kaziranga National Park
Indus river system: 2. Pobitora wildlife Sanctuary
1. Jhelum river finds its origin in India itself. 3. Nameri National Park
2. Ravi river finds its origin in Himachal Pradesh. Select the correct Ans using the codes given below:
3. Indus drains the UTs of Jammu Kashmir and a) 1 and 2 only
Ladakh. b) 2 and 3 only
Select the correct Ans using the codes given below: c) 1 only
a) 1 and 2 only d) 2 only
b) 2 and 3 only
c) 1 and 3 only Q.15) Which of the following parameters are used by
d) 1, 2, and 3 the Indian Meteorological Department (IMD)
for the official declaration of the onset of Monsoon
Q.11) Consider the following statements: in India?
1. Joshimath is associated with the river Bhagirathi. 1. Rainfall reported over various weather stations
2. The length of Ganga is least in the state of located in Kerala only
Uttarakhand. 2. Atmospheric Pressure exerted by the Westerlies
3. The Ganga river system is entirely composed of 3. Outwave Longwave Radiation
rivers of Himalayan origin. Choose the correct Ans using the code given below:
Select the correct Ans using the codes given below: a) 1 and 2 only
a) 1 and 3 only b) 2 and 3 only
b) 2 only c) 1 and 3 only
c) 3 only d) 1,2 and 3
d) 1 and 2 only
Q.16) In the context of India’s climate, which of the
Q.12) Consider the following pairs: following correctly describes the phenomenon of
1. Rivers Source “Monsoon Break”?
2. Alaknanda Satopanth Glacier a) A rain deficient season where the total amount of
3. Yamuna Bandarpunch rainfall received is drastically less than the Long
e

4. Son Rajmahal hills Period Average.


in

5. Kosi Dhaulagiri b) A scenario where a cyclone induced low pressure


nl

Which of the above pairs are correctly matched? delays the onset of the Monsoon in India.
l.o

a) 1 and 4 only c) The simultaneous occurrence of the gradual


b) 2 and 3 only withdrawal of the Southwest Monsoons and the
ria

c) 1 and 2 only gradual onset of the North East Monsoons.


e

d) 2 and 4 only d) A sudden but sharp decrease in rainfall over most


at

parts of the country during the peak Monsoon


cm

Q.13) Consider the following statements: season.


ps
.u

PRAYAS TEST 9 2
w
w
w
Contact us : info@onlyias.com

OnlyIAS Nothing Else Visit : dpp.onlyias.in


Contact : +91-7007 931 912

Q.17) Which of the following would have happened 2. Inter season variability
if the Himalayas did not exist? 3. Western Disturbances
a) There would be no North Eastern Monsoon 4. Intra season variability
season experienced in India. 5. Sudden withdrawal
b) The onset of monsoon would have taken place in Choose the correct Ans using the code given below:
the winter season. a) 1,2 and 4 only
c) The Indian subcontinent would have recorded a b) 3,4 and 5 only
lower Long Period Average of rainfall received. c) 1,2,3 and 4 only
d) The Thar desert would have received very high d) 1,2,3,4 and 5
rainfall every year.
Q.21) Consider the following statements:
Q.18) Which of the following conditions are taken 1. The El-Nino is generally known to suppress
into account by the Earth System Science monsoon rains in India.
Organisation (ESSO) and the India Meteorological 2. The La-Nina has a negative impact on rainfall
Department (IMD) while making forecasts about the associated with the Northeast monsoon.
monsoon in India? 3. Heavy snowfall in Tibetan Plateau in the winter
1. Mean Sea Level pressure in east Asia season can immensely increase the rainfall
2. The surface air temperature over northwest received in India during the monsoon season.
Europe Which of the above statements is/are correct?
3. Sea Surface Temperature (SST) of the equatorial a) 1 only
South Indian Ocean b) 1 and 2 only
4. SST gradient between the North Atlantic and the c) 1 and 3 only
North Pacific OceAns d) 2 and 3 only
Choose the Correct Ans using the code given below:
a) 1,2 and 4 only Q.22) Which of the following statements regarding
b) 2,3 and 4 only red soils of India are correct?
c) 1 and 3 only 1. They are generally poor in nitrogen and humus.
d) 1,2,3 and 4 2. They are formed due to weathering of ancient
crystalline granitic rocks.
3. Rice and pulses can be grown on these soils and
Q.19) With reference to the Indian Ocean Dipole cotton cannot be grown on these soils.
(IOD), Consider the following statements: 4. They occupy roughly 40% of the total geographical
1. It is characterized by an oscillation of sea surface area of India.
temperatures in which the Western Indian Ocean Choose the correct Ans using the code given below:
becomes alternately warmer and colder than the a) 1 and 2 only
Eastern part of the Pacific Ocean. b) 1,3 and 4 only
2. Unlike an El-Nino, the IOD is a periodically c) 2 and 3 only
recurring event and can be predicted. d) 1,2 and 4 only
3. It is possible for IOD to cause floods in India and
e

bushfires in Australia in the same year. Q.23) “This deciduous tree, also called red silk-cotton
in

Which of the above statements are correct ? tree, is used across cultures and geographical
nl

a) 1 and 2 only boundaries for its ethnomedicinal properties. A


l.o

b) 2 and 3 only biosphere reserve in India derives its name from this
c) 3 only tree because of its abundance inside the reserve”
ria

d) 1,2 and 3 The tree referred to in the above excerpt is


e

a) Kusum
at

Q.20) Which of the following are characteristic b) Shisham


cm

features of the Indian Monsoons? c) Semul


1. Sudden Burst d) Hurra
ps
.u

PRAYAS TEST 9 3
w
w
w
Contact us : info@onlyias.com

OnlyIAS Nothing Else Visit : dpp.onlyias.in


Contact : +91-7007 931 912

Q.24) Consider the following states: Choose the correct Ans using the code given below:
1. Haryana a) 1 and 2 only
2. Rajasthan b) 3 only
3. Gujrat c) 2 and 3 only
4. Andhra Pradesh d) 1 and 3 only
Where will you naturally find “Thorny Scrub
Forests”? Q.29) Which of the following states of India have
a) 1,2 and 3 only Recoverable Reserves of Natural Gas?
b) 2 and 3 only 1. Nagaland
c) 1 and 4 only 2. Arunachal Pradesh
d) 1,2,3 and 4 3. Tamil nadu
4. Madhya Pradesh
Q.25) Dolomite powder is sometimes applied in Choose the correct Ans using the code given below:
agricultural lands. The purpose of applying it is to a) 1,2 and 3 only
a) Increase pH of the soil b) 2,3 and 4 only
b) Lower pH of the soil c) 1 and 4 only
c) Enrich biota in the soil d) 1,2,3 and 4
d) Increase nitrogen content in the soil
Q.30) Consider the following Minerals:
Q.26) Consider the following National parks: 1. Manganese
1. Campbell Bay National Park 2. Nickel
2. Gugamal National Park 3. Chromite
3. Dandeli Anshi National Park 4. Dolomite
4. Mukundara Hills National Park 5. Bauxite
In which of the above National parks will you find 6. Mica
Ebony and Mahogany trees naturally ? Which of the above minerals are metallic in nature?
a) 1 and 2 only a) 1,2,5 and 6 only
b) 1 and 3 only b) 1,2,3 and 5 only
c) 2 and 4 only c) 2,3,4 and 6 only
d) 1,3 and 4 only d) 1,3,4,5 and 6 only

Q.27) Which of the following methods is the most Q.31) ‘Limonite’, Siderite’ and ‘Lodestone’,
appropriate for checking soil erosion on Coastal and sometimes mentioned in the context of mining in
arid areas? India, are inferior ores of
a) Contour Bunding a) Copper
b) Shelterbelts b) Manganese
c) Contour Ploughing c) Iron
d) Check Dams d) Bauxite
e

Q.28) With reference to Tropical Moist Deciduous Q.32) Which of the following minerals are found
in

Forests, which of the following statements is/are naturally in the State of Odisha ?
nl

correct ? 1. Iron ore


l.o

1. The Trees shed leaves their leaves annually during 2. Manganese


the autumn season. 3. Chromium
ria

2. In India, the total area covered by these forests is 4. Bauxite


e

much more than the area covered by Littoral and Choose the correct Ans using the code given below:
at

Swamp Forests. a) 1 and 2 only


cm

3. In India, both South West monsoon and NorthEast b) 2,3 and 4 only
Monsoon influence their growth. c) 1,3 and 4 only
ps
.u

PRAYAS TEST 9 4
w
w
w
Contact us : info@onlyias.com

OnlyIAS Nothing Else Visit : dpp.onlyias.in


Contact : +91-7007 931 912

d) 1,2,3 and 4 b) 2 only


c) Both 1 and 2
Q.33) Consider the following statements: d) Neither 1 nor 2
1. Diamonds fields exist in more than one state in
India. Q.37) Consider the following statements about
2. Commercial diamond mining is done in more than Western Disturbances:
one state in India. 1. Western Disturbances move towards northwest
3. Till recently, the Diamond cutting industry in India India under the influence of Tropical Easterly Jet
was dependent on imports of uncut diamonds, Stream.
but India has now become self-sufficient in 2. They usually get moisture from Mediterranean
Diamond production. and Atlantic Ocean.
Which of the above statements is/are correct ? 3. High pressure over Ukraine helps in formation of
a) 1 only Western Disturbances.
b) 1 and 2 only Which of the above statement(s) is/are correct?
c) 2 and 3 only a) 1 only
d) 1 and 3 b) 1 and 2 only
c) 2 and 3 only
Q.34) Consider the following statements: d) 3 only
1. Cuddapah is known for its Limestone deposits
2. Subarnarekha River is known for its gold placer Q.38) Which of the following statements with
deposits. reference to Omega Block is correct?
3. The Maikal Range is known for its Bauxite a) It is a low-pressure pattern that blocks and diverts
deposits. jet streams.
Which of the above statements are correct? b) Because of the Omega block, cold air from
a) 1 and 2 only southern Europe and Africa can be pulled north.
b) 2 and 3 only c) Formation of Omega block leads to formation of
c) 1 and 3 only Spanish plume.
d) 1,2 and 3 d) None of the above

Q.35) With reference to Monazite, Consider the Q.39) Consider the following statements about
following statements: effects of El-Nino:
1. In India, Monazite is found in beach sand only. 1. It results in dry conditions and forest fires in
2. In India, Monazite is exploited for extraction of Australia and Indonesia.
rare earths and thorium. 2. Fewer hurricanes over the Atlantic Ocean, the
3. Tamil Nadu and Andhra Pradesh have multiple Caribbean Sea and the Gulf of Mexico.
deposits of monazite 3. Bleaching of corals in the Lakshadweep islands.
4. Odisha has no monazite deposits. Which of the above statements are correct?
Which of the above statements is/are correct? a) 1 and 2 only
a) 1,2 and 4 only b) 2 and 3 only
e

b) 2 and 3 only c) 1 and 3 only


in

c) 1 and 3 only d) 1, 2 and 3


nl

d) 2,3 and 4 only


l.o

Q.40) Consider the following statements about


Q.36) Consider the following statements: walker circulation:
ria

1. The eye of a tropical cyclone is a region of calm 1. Walker circulation is formed due to development
e

and subsiding warm air. of pressure gradient from west to east in the
at

2. Tropical cyclones generally do not have lightning. equatorial Pacific Ocean.


cm

Which of the above statement(s) is/are correct? 2. Years of walker circulation are beneficial for the
a) 1 only people of Peru.
ps
.u

PRAYAS TEST 9 5
w
w
w
Contact us : info@onlyias.com

OnlyIAS Nothing Else Visit : dpp.onlyias.in


Contact : +91-7007 931 912

3. La-Nina is a strengthened phase of walker 3. Desert climates are favourable around the
circulation. subtropical high-pressure belts.
Which of the above statements are correct? Select the correct Ans using the codes given below:
a) 1 and 2 only a) 1 and 2 only
b) 2 and 3 only b) 2 and 3 only
c) 1 and 3 only c) 3 only
d) 1, 2 and 3 d) 1, 2, and 3

Q.41) Consider the following statements: Q.45) Consider the following statements:
1. Equator divides the Llanos and Campos grasslands 1. Crop growth is faster in winters in the
of Savanna. Mediterranean regions due to winter rains.
2. Westerlies are dominant in shaping the climate of 2. Balearic Islands are situated at the center of the
Savanna regions. Mediterranean Sea.
Select the correct Ans using the codes given below: Select the correct Ans using the codes given below:
a) 1 only a) 1 only
b) 2 only b) 2 only
c) Both 1 and 2 c) Both 1 and 2
d) Neither 1 nor 2 d) Neither 1 nor 2

Q.42) Consider the following events: Q.46) Consider the following statements:
1. Sunspot cycle 1. The temperate grasslands of South America are
2. Volcanism devoid of marine influence.
3. Aerosol Radiative Forcing 2. Steppes in the Northern Hemisphere experience
4. Milankovitch oscillations mild winters due to continentality.
5. Glacial Lake sediments Select the correct Ans using the codes given below:
Which of the above are the astronomical causes for a) 1 only
climate change? b) 2 only
a) 1, 2, and 5 only c) Both 1 and 2
b) 1, 2, and 5 only d) Neither 1 nor 2
c) 1, 3, and 5 only
d) 1, 2, and 4 only Q.47) Consider the following statements:
1. Valley of flowers national park forms the core
Q.43) Consider the following statements: area of the Nanda Devi Biosphere reserve.
1. Rice cultivation accounts for maximum 2. Glacial retreats are an important cause of the
Greenhouse gases emissions from the agricultural Glacial Lake Outburst floods.
sector. 3. The National Mission for sustaining Himalayan
2. Areas with monsoonal climate largely depend Ecosystem is under the Ministry of Environment,
upon agriculture for livelihood. Forests, and Climate Change.
Select the correct Ans using the codes given below: Select the correct Ans using the codes given below:
e

a) 1 only a) 1 and 2 only


in

b) 2 only b) 2 and 3 only


nl

c) Both 1 and 2 c) 1 and 3 only


l.o

d) Neither 1 nor 2 d) 1, 2, and 3


ria

Q.44) Consider the following statements: Q.48) Consider the following pairs:
e

1. Deserts in South America are formed only Grasslands Country


at

because of Continentality. 1. Llanos Ecuador


cm

2. Western boundaries of most African deserts are 2. Pampas Uruguay


flanked by cold ocean currents. 3. Pustaz Hungary
ps
.u

PRAYAS TEST 9 6
w
w
w
Contact us : info@onlyias.com

OnlyIAS Nothing Else Visit : dpp.onlyias.in


Contact : +91-7007 931 912

4. Canterbury Britain a) 1 and 2 only


Select the correct Ans using the codes given below: b) 2 and 3 only
a) 1 and 2 only c) 1 and 3 only
b) 2 and 3 only d) 1, 2, and 3
c) 1 and 4 only
d) 1 and 3 only Q.53) Consider the following statements:
1. Arctic Council is a non-governmental organization
Q.49) Consider the following statements: aimed at environmental protection.
1. Temperate Monsoonal climates are mostly found 2. Himadri is India’s research base deployed in
in China. Norway for the study of the region.
2. Gulf type climate is most prominent along the Select the correct Ans using the codes given below:
coast of Yemen. a) 1 only
3. The grasslands in Savanna are distinctive due to b) 2 only
the absence of trees. c) Both 1 and 2
Select the correct Ans using the codes given below: d) Neither 1 nor 2
a) 1 only
b) 3 only Q.54) Consider the following statements:
c) 1 and 2 only 1. Greenhouse gases are capable of absorbing short-
d) 2 and 3 only wave radiations of the sun.
2. Both the Kyoto Protocol and Paris Agreement
Q.50) Consider the following statements: impose legally binding obligations.
1. The whole of Tundra region is characterized by 3. India aims to have 70% of non-fossil fuel-based
permanent snow cover. electricity by 2030.
2. MOSAiC project is designed for the study of the Select the correct Ans using the codes given below:
Antarctica climate. a) 1 and 2 only
Select the Ans using the codes given below: b) 2 only
a) 1 only c) 1 and 3 only
b) 2only d) 3 only
c) Both 1 and 2
d) Neither 1 nor 2 Q.55) Consider the following:
1. Neem coating of urea
Q.51) Consider the following statements: 2. Soil Health card
1. Coniferous trees are totally absent below the 3. Organic fertilizers
Equator due to the narrowness of continents. Which of the above is/are useful in reducing the
2. Pine, Fir, Spruce are the major trees found in the emission of nitrous oxide(N2O)?
Tundra regions. a) 1 and 2 only
3. Laurentian climate finds presence in the Northern b) 1, 2, and 3
Hemisphere only. c) 3 only
Select the incorrect Ans using the codes given below: d) 2 and 3 only
e

a) 1 and 3 only
in

b) 3 only Q.56) This region is far ahead of many other areas on


nl

c) 1 and 2 only account of their unprecedented industrial


l.o

d) 1, 2, and 3 advancement. Cyclonic disturbances ensure


adequate rainfall to these areas throughout the year.
ria

Q.52) Consider the following pairs: The temperature lies between 5 to 15 degree C
e

1. Land of big games Savanna throughout the year. The deciduous trees dominate
at

2. Granaries of the world Prairies the region and shed their leaves mainly in autumn.
cm

3. Cold pole of the Earth Siberia Which of the following climatic regions is being
Which of the above pair(s) is/are correctly matched? referred to above?
ps
.u

PRAYAS TEST 9 7
w
w
w
Contact us : info@onlyias.com

OnlyIAS Nothing Else Visit : dpp.onlyias.in


Contact : +91-7007 931 912

a) Mediterranean climate a) 1 and 2 only


b) Tropical Monsoon climate b) 2 and 4 only
c) British Climate c) 1 and 3 only
d) Steppe’s grasslands d) 1 and 4 only

Q.57) Consider the following statements: Q.61) Which of the following statements is most
1. Steppes are found in the areas around the Black appropriate regarding passes in India?
Sea and the Caspian Sea. a) The Lampiya dura pass is located to the south of
2. Thermal expAnsion of the ocean water is the main Lipulekh pass.
contributing factor to the rise in sea level. b) Mana pass is in the Khangchendzonga biosphere
Select the correct Ans using the codes given below: reserve.
a) 1 only c) Palghat pass is situated between the Nilgiris and
b) 2 only the Cardamom Hills.
c) Both 1 and 2 d) Jawahar Tunnel is also known as the Banihal pass.
d) Neither 1 nor 2
Q.62) Consider the following statements:
Q.58) Consider the following statements: 1. Bhabar areas are more suitable for crop
1. Lake Priyadarshini is built by India around Maitri cultivation than Terai.
research base, Antarctica. 2. Khadar sediments are deposited near the flood
2. Polar Vortex and Polar Stratospheric clouds are plains.
directly responsible for Ozone depletion. Select the correct Ans using the codes given below:
3. Kigali Amendment to the Montreal Protocol is a) 1 only
legally binding in nature. b) 2 only
Select the correct Ans using the codes given below: c) Both 1 and 2
a) 1 and 2 only d) Neither 1 nor 2
b) 1 and 3 only
c) 2 only Q.63) Consider the following statements:
d) 1,2 and 3 1. Bhabar, Tarai, Khadar, and Bhangar together
constitute alluvial plains.
Q.59) Consider the following statements: 2. Most of the Brahmaputra plains lie in the state of
1. The Indian Standard Meridian (ISD) cuts across the Arunachal Pradesh.
centre of Andhra Pradesh. Select the correct Ans from the codes given below:
2. The smallest part of ISD falls in the state of a) 1 only
Madhya Pradesh. b) 2 only
Select the correct Ans using the codes given below: c) Both 1 and 2
a) 1 only d) Neither 1 nor 2
b) 2 only
c) Both 1 and 2 Q.64) Consider the following statements:
d) Neither 1 nor 2 1. The elevation of western ghats increases
e

southwards.
in

Q.60) Consider the following statements: 2. Eastern ghats are continuous with lower
nl

1. Molassis basin is an important feature in the state elevation.


l.o

of Mizoram. 3. Bundelkhand forms the upper part of the Deccan


2. Mahabharat range is located between the Indo- Plateau.
ria

Nepal border. Select the correct Ans from the codes given below:
e

3. Aksai Chin forms a part of the Karakoram range. a) 1 and 2 only


at

4. Nubra valley is accessed through the Khardung La b) 1 only


cm

pass. c) 2 and 3 only


Select the correct Ans using the codes given below: d) 1, 2 and 3
ps
.u

PRAYAS TEST 9 8
w
w
w
Contact us : info@onlyias.com

OnlyIAS Nothing Else Visit : dpp.onlyias.in


Contact : +91-7007 931 912

Q.65) Consider the following statements: d) 1, 2 and 3


1. All the rivers of the Central Highlands originate in
the Vindhya and Satpura ranges. Q.69) Which of the following statements is most
2. Malda fault exists between the Malwa Plateau appropriate regarding the Islands of India?
and the Shillong Plateau. a) The entire Island of Lakshadweep lies below the
Select the incorrect Ans using the codes given below: 12-degree channel.
a) 1 only b) Duncan Passage is situated between Swaraj
b) 2 only dweep and the Shaheed dweep.
c) Both 1 and 2 c) Agatti Island is situated to the north of the capital
d) Neither 1 nor 2 Kavaratti.
d) The highest peak of Andaman, Saddle Peak is
Q.66) Consider the following ranges/mountains: located in the South Andaman.
1. Harishchandra
2. Nallamala Q.70) Consider the following statements regarding
3. Dandakaranya Andaman and Nicobar Islands:
4. Ramgarh 1. These islands are an extension of the Arakan
5. Malya giri Yoma range.
6. Garhjat 2. Narcondam Island in the Andaman is a dormant
Arrange the above ranges in the south-north volcano.
direction: 3. Mt. Thullier is the highest peak of the Nicobar
a) 2-3-1-6-5-4 group of Islands.
b) 2-1-3-4-6-5 Select the correct Ans using the codes given below:
c) 2-1-3-5-6-4 a) 1 and 3 only
d) 1-2-4-3-6-5 b) 1 and 2 only
c) 2 and 3 only
Q.67) Consider the following statements regarding d) 1, 2 and 3
the Indian desert:
1. Absence of marine deposits indicates that the Q.71) Consider the following statements regarding
region was a part of the Peninsular plateau. Lakshadweep:
2. This area shows opposing slopes in the Northern 1. Adroit is the largest group of Islands in
and the Southern parts. Lakshadweep.
Select the correct Ans using the codes given below: 2. Minicoy Island is separated from the rest of the
a) 1 only island through a 9-degree channel.
b) 2 only Select the correct Ans using the codes given below:
c) Both 1 and 2 a) 1 only
d) Neither 1 nor 2 b) 2 only
c) Both 1 and 2
Q.68) Consider the following statements regarding d) Neither 1 nor 2
the Indian coasts:
e

1. Natural harbors and ports are a characteristic Q.72) Consider the following statements:
in

feature of the emergent coasts. 1. Catchment area of a river is located near the
nl

2. Gujarat has the maximum coastline amongst all source of the river.
l.o

the States/UTs of India. 2. Drainage basin of a river is formed by the


3. Northern Circars are the region lying between mainstream excluding the tributaries.
ria

Mahanadi and Krishna 3. Drainage basin confined to a large geographical


e

Select the correct Ans using the codes given below: area forms the watershed.
at

a) 1 and 2 only Select the incorrect Ans using the codes given below:
cm

b) 3 only a) 1 and 2 only


c) 2 and 3 only b) 2 and 3 only
ps
.u

PRAYAS TEST 9 9
w
w
w
Contact us : info@onlyias.com

OnlyIAS Nothing Else Visit : dpp.onlyias.in


Contact : +91-7007 931 912

c) 1 and 3 only 1. Vamsadhara is a west flowing river forming an


d) 1 only estuary.
2. Andhra Pradesh, Karnataka, Kerala forms the
Q.73) Consider the following rivers: drainage basin of Pennar river.
1. Sindh Select the incorrect Ans using the codes given below:
2. Son a) 1 only
3. Mayurakshi b) 2 only
4. Ken c) Both 1 and 2
Arrange the rivers above in the west to east d) Neither 1 nor 2
direction:
a) 3-1-4-2 Q.78) Consider the following statements:
b) 1-4-2-3 1. Periyar is the largest river of Kerala.
c) 3-4-1-2 2. Vembanad lake is an example of lagoon lake.
d) 1-2-4-3 Select the correct Ans using the codes given below:
a) 1 only
Q.74) Consider the following statements: b) 2 only
1. Antecedent streams maintain their course despite c) Both 1 and 2
the upliftment of the surrounding surface. d) Neither 1 nor 2
2. Consequent streams flow according to the slope
of the adjoining surface. Q.79) the Consider following statements:
Select the correct Ans using the codes given below: 1. The discharge of the Himalayan rivers is constant
a) 1 only throughout the year.
b) 2 only 2. Harike wetland is situated at the confluence of
c) Both 1 and 2 Satluj and Beas.
d) Neither 1 nor 2 3. Verinag spring in Himachal Pradesh is the origin of
Jhelum.
Q.75) Consider the following pairs: Select the correct Ans using the codes given below:
Rivers State of Origin a) 1 and 2 only
1. Mahanadi Odisha b) 2 only
2. Kaveri Kerala c) 2 and 3 only
3. Tapi Gujarat d) 1, 2 and 3
4. Krishna Karnataka
Which of the above pairs is/are correctly matched? Q.80) Consider the following statements:
a) 1, 2 and 4 only 1. Renuka Lake is an artificial freshwater lake in
b) 1, 3 and 4 only Himachal Pradesh.
c) 3 and 4 only 2. It is the only Ramsar wetland site of the state.
d) None of the above Select the correct Ans using the codes given below:
a) 1 only
Q.76) Consider the following statements: b) 2 only
e

1. River Godavari drains five states of India. c) Both 1 and 2


in

2. River Mandovi is known for delta formation. d) Neither 1 nor 2


nl

Select the correct Ans using the codes given below:


l.o

a) 1 only Q.81) Consider the following rivers:


b) 2 only 1. Raidak
ria

c) Both 1 and 2 2. Rind


e

d) Neither 1 nor 2 3. Sengar


at

4. Sindh
cm

Q.77) Consider the following statements: 5. Son


Which of the above is/are the tributaries of Yamuna?
ps
.u

PRAYAS TEST 9 10
w
w
w
Contact us : info@onlyias.com

OnlyIAS Nothing Else Visit : dpp.onlyias.in


Contact : +91-7007 931 912

a) 1,2, and 4 c) 1 and 2 only


b) 2, 3 and 4 d) 1,2 and 3
c) 3, 4, and 5
d) 1, 4, and 5 Q.86) Which of the following statements
is not correct with reference to Cloudbursts ?
Q.82) Consider the following statements: a) The Specific location and time of a cloudburst can
1. Rivers originating from the Amarkantak range are be predicted only after the genesis of a
an example of centripetal drainage. thunderstorm has already begun.
2. Rivers Narmada and Son flow in different b) These systems have a shorter life span whereas
directions despite the same source of origin. large scale systems like tropical cyclones have a
Select the correct Ans using the codes given below: longer life span.
a) 1 only c) A cloudburst can occur anytime and at any place
b) 2 only which is affected by convective weather systems.
c) Both 1 and 2 d) The India Meteorological Department labels
d) Neither 1 nor 2 rainfall over 200 mm per hour as cloudburst.

Q.83) Consider the following south Indian rivers:


1. Penganga Q.87) With reference to cyclones in India, consider
2. Penneru the following statements:
3. Tungabhadra 1. Historically in India, more cyclones have impacted
4. Manjira the Eastern coast in comparison to the Western
The correct arrangement of these rivers in the North- Coast.
South direction is: 2. Historically in India, more cyclones have occurred
a) 1-3-4-2 in the post monsoon season in comparison to the
b) 1-4-3-2 Pre-monsoon Season.
c) 4-1-3-2 3. All cyclones that hit the Indian coast originate in
d) 4-1-2-3 the Indian Ocean.
Which of the above statements is/are correct ?
Q.84) Consider the following statements: a) 1 and 2 only
a) Majority of Indian rivers flow through a rift valley. b) 2 and 3 only
b) Lacustrine deposits have been found from the c) 1 and 3 only
Shivalik. d) 1,2 and 3
c) There is a sharp tilt of the Indian peninsula from
west to east. Q.88) With reference to Madden Julian oscillations
d) Most of Tapi’s drainage basin lies in the state of (MJO), consider the following statements:
Gujarat. 1. Unlike a standing pattern like the El Niño, the
Madden–Julian oscillation is a traveling pattern
that propagates eastward above the warm parts
Q.85) With reference to Western of the Indian and Pacific oceAns.
e

disturbances, consider the following statements: 2. A MJO cycle can last anywhere between 120 to
in

1. These are weak tropical cyclones that originate 150 days.


nl

over the Mediterranean sea. 3. A MJO can completely nullify the impact of El-
l.o

2. Jetstreams play an important role in driving these Nino on monsoon rains in India.
high pressure systems into India. Which of the above statements is/are correct ?
ria

3. The rainfall that occurs due to these disturbances a) 1 only


e

is known to be beneficial for the Rabi crops. b) 1 and 2 only


at

Which of the above statements is/are incorrect ? c) 1 and 3 only


cm

a) 1 only d) 2 and 3 only


b) 2 only
ps
.u

PRAYAS TEST 9 11
w
w
w
Contact us : info@onlyias.com

OnlyIAS Nothing Else Visit : dpp.onlyias.in


Contact : +91-7007 931 912

Q.89) With reference to heatwaves in India, Which of b) 2,3 and 5 only


the following statements is not correct ? c) 2,3 and 4 only
a) In General, heatwaves are defined with reference d) 1,2,4 and 5 only
to the normal maximum temperature of a Q.93) With reference to the Andaman and Nicobar
particular region Islands, which of the following statements is/are
b) Most heat waves in India are formed over the incorrect?
Peninsular region especially on the leeward side 1. The Andaman Islands are the extension of the
of the Western ghats Arkan yoma mountain ranges of Myanmar.
c) Inflow of Western disturbances in Northern India 2. Andaman Islands are separated from Nicobar
can influence the occurrence of heat waves in Islands by 9-degree channel
Central India 3. Saddle peak is the highest in Great Nicobar and
d) A cyclone hitting the cost of Odisha can trigger a Mt.Thuillier is the highest peak in North Andaman
heatwave in Punjab 4. Ross island is the largest island in the union
territory
Q.90) According to the Indian Meteorological Select the correct Ans using the codes given below:
Department, in which of the following states/union a) Only 1
territories does the South West Monsoon arrive first b) 1 and 2 only
? c) 2,3, and 4 only
a) Kerala d) 1,2,3, and 4
b) Lakshadweep
c) Andaman and Nicobar Islands Q.94) Consider the following pairs:
d) Tripura River Tributary
1. Son South Koel
Q.91) Consider the following statements: 2. Brahmaputra Sankosh
1. Norwesters are violent storms that generally 3. Krishna Musi
occur just before sunset or just a few hours after 4. Mahanadi Kangsabati
it. Select the correct pair/pairs using the code given
2. In India, these storms originate over Chotanagpur below:
Plateau only. a) 1 only
3. In India, the rainfall caused by Norwesters is b) 2 only
deemed harmful for the tea cultivated in Assam c) 2 and 3 only
and the jute cultivated in West Bengal. d) 2, 3 and 4 only
Which of the above statements is/are correct ?
a) 1 only Q.95) With reference to the Chambal river, Consider
b) 1 and 2 only the following statements.
c) 2 and 3 only 1. It originates along the Son river in Amarkantak
d) 1 and 3 only Plateau of Madhya Pradesh.
2. Chambai is the chief tributary of the Ganga river
Q.92) In India, The Western Ghats receive rain from which flows South to North direction.
e

which of the following sources on an annual basis? 3. Banas is the only right bank tributary of the
in

1. Bay of Bengal Branch of South West Monsoon Chambal river.


nl

2. Cyclones and depressions that originate in the Bay 4. Chambal is famous for badland topography called
l.o

of Bengal Chambal Ravines.


3. Cyclones and depressions that originate in the Which of the statements given above is/are NOT
ria

Arabian Sea correct?


e

4. Western Disturbances a) 1, 2 and 3 only


at

5. North East Monsoons b) 2, 3 and 4 only


cm

Select the correct Ans using the code given below: c) 1, 3 and 4 only
a) 1,3 and 5 only d) 1, 2, 3 and 4
ps
.u

PRAYAS TEST 9 12
w
w
w
Contact us : info@onlyias.com

OnlyIAS Nothing Else Visit : dpp.onlyias.in


Contact : +91-7007 931 912

Q.96) Consider the following pairs : Which of the statements given above is/are NOT
River Origin correct?
1. Alaknanda - Satopanth Glacier a) 1 only
2. Ghaghara - Mapchachungo Glacier b) 1 and 2 only
3. Teesta Kangto Glacier c) 2 only
4. Sarda Cho Lamu Glacier d) 1 and 3 only
5. Brahmaputra Chemayungdung Glacier
Which of the pairs given above are correct? Q.100) With reference to the Indian
a) 1 and 5 only Monsoon, Consider the following statements
b) 2 and 3 only 1. North East Monsoon winds blow during the
c) 1, 2 and 5 only months of October to December and are confined
d) 1, 2, 3 and 5 only to the north-east region of India.
2. North- East Monsoon Winds characterized by
Q.97) Which of the following rivers do not form a Oppressive heat and humidity whereas Retreating
radial drainage pattern? monsoon winds creates pleasant seasons with low
1. Johilla River temperature and low humidity.
2. Son River Which of the statements given above is/are correct?
3. Machhu River a) 1 only
4. Vamsadhara River b) 2 only
5. Luni river c) Both 1 and 2
Select the correct Ans using the code given below: d) Neither 1 nor 2
a) 1 and 2 only
b) 2, 3 and 5 only
c) 1, 3 and 4 only
d) 4 and 5 only

Q.98) Consider the following statements.


1. Tamil Nadu Coast (South-East) receives heavy
rains from the Bay of Bengal Branch of Indian
Monsoon.
2. North-West region of India including Punjab,
Haryana receives winter rainfall due to Tropical
Easterly Jet Stream.
Which of the statements given above is/are correct
?
a) 1 only
b) 2 only
c) Both 1 and 2
d) Neither 1 nor 2
e
in

Q.99) Consider the following statements:


nl

1. Karnataka receives more rainfall than


l.o

Maharashtra from the Arabian sea branch of


Indian South West Monsoon.
ria

2. Chotanagpur Plateau receives miniscule rainfall


e

from the Bay of Bengal branch of Indian South


at

West Monsoon.
cm

3. Tamil Nadu coast is situated parallel to the Bay of


Bengal Branch of South-west monsoon.
ps
.u

PRAYAS TEST 9 13
w
w
w
Contact us : info@onlyias.com

OnlyIAS Nothing Else Visit : dpp.onlyias.in


Contact : +91-7007 931 912

Q.1) Ans: d Q.4) Ans: d


Exp: Exp:
 Statement 1 is incorrect: Manipur and  Statement 1 is incorrect: The Himalayas in the
Nagaland are the states that share borders with northwestern region are aligned in the Northwest
Myanmar only while Mizoram shares borders to southeast direction while the Himalayas in the
with both Myanmar and Bangladesh. Northeastern region are not continuous and
 Statement 2 is incorrect: Sikkim and Tripura are aligned in various directions.
the states that form international boundaries on  Darjeeling and Sikkim in the Northeast direction.
the three sides. Sikkim is bordered by Tibet,  Arunachal Pradesh in the Southwest to Northeast
Nepal, and Bhutan whereas Tripura is surrounded direction.
by Bangladesh from three sides.  Nagaland, Manipur, and Mizoram in the North
 Statement 3 is incorrect: West Bengal forms the south direction.
longest international boundary with Bangladesh.  Statement 2 is incorrect: Meghalaya and Karbi
Besides, West Bengal, Meghalaya, Assam, Tripura, Anglong hills in Assam are an extension of the
and Mizoram also share a boundary with Peninsular Plateau. They are a part of the
Bangladesh. Northeastern plateau and not the Deccan Plateau.
 Statement 3 is incorrect: Darjeeling, Sikkim and
the Arunachal Himalayas are different from the
Q.2) Ans: b rest as there is an absence of Shivalik in these
Exp: areas. In place of Shivalik, duar formations are
 Statement 1 is incorrect: The latitude of Jaipur is found in these areas which are used for the
26.914 degrees North while that of Lucknow is cultivation of the plantation crops. Hence, duar
26.8467 degrees North. Hence, Jaipur is to the are different from Shivaliks and not an extension
North of Lucknow. of it.
 Statement 2 is correct: The longitude of
Hyderabad is 78.4867degrees East, Kurnool is Q.5) Ans: b
78.0373 degrees East, and Amravati is 77.7523 Exp:
degrees East, Hence, Hyderabad lies between  Statement B is correct: Saramati Peak is situated
Kurnool and Amravati. in the state of Nagaland whereas Nokrek is
 Statement 3 is correct: Palghar is situated to the situated in the state of Meghalaya. Annapurna
north of Visakhapatnam. The latitude of Palghar is and Makalu are a part of Nepal, where Makalu is
19.6967 degrees North, while that of to the east of Annapurna. Kamet peak and Nanda
Vishakhapatnam is 17.6868 degrees North. Devi lie in the state of Uttarakhand with Nanda
devi to the east of Kamet. Hence, the correct
order from the east to west direction is Saramati,
Q.3) Ans: d Nokrek, Makalu, Annapurna, Nanda Devi, and
Exp: Kamet.
 Statement 1 is incorrect: Indira col is the
northernmost extreme point of India. It is situated Q.6) Ans: a
e

near Siachen Glacier in Ladakh. Whereas Indira Exp:


in

point is in the Great Nicobar at the southern point  Statement 1 is correct: Aravallis are the residual
nl

of India. Kanyakumari is the southernmost mountains in the western India. They are spread
l.o

extreme of India in the Indian mainland. across the four states/UTs i.e., Rajasthan,
 Statement 2 is incorrect: Kibithu in Arunachal
ria

Haryana, Gujarat, Delhi. Its highest peak is Guru


Pradesh is the eastern most extreme point of Shikhar located in Mt. Abu.
e

India.  Statement 2 is incorrect: The Indian Himalayan


at

 Statement 3 is correct: Guhar Moti in Gujarat is Range extends to the states Himachal Pradesh,
cm

the western most extreme point of India. Uttarakhand, Sikkim, West Bengal, Assam,
Meghalaya, Assam, Arunachal Pradesh, Nagaland,
ps
.u

PRAYAS TEST 9 14
w
w
w
Contact us : info@onlyias.com

OnlyIAS Nothing Else Visit : dpp.onlyias.in


Contact : +91-7007 931 912

Manipur, Mizoram, and Tripura. It also extends to  Statement 1 is correct: The downward depression
the UTs of Ladakh and Jammu and Kashmir. created due to the Malda gap between the
Tripura is a Himalayan state, and its highest peak Rajmahal hills and the Meghalaya plateau caused
is Betling Shib in the Jampui range. the Ganga and Brahmaputra rivers to drain to the
 Statement 3 is correct: Tree line is the edge of the Bay of Bengal. Hence, it played a role in the
habitat at which trees are found. Beyond the tree evolution of the Himalayan rivers.
line, there is no possibility of finding a tree. Tree  Statement 2 is correct: Potwar Plateau or the
line of the western Himalayas is lower than that of Delhi ridge acted as a water divide between the
the Eastern Himalayas. Indus and the Ganga river system.
 Statement 3 is correct: The upliftment of western
Q.7) Ans: d Himalayas also played a role in the creation of the
Exp: present-day Himalayan drainage system which
 Statement 1 is incorrect: Karewas are the thick comprises the Indus, Ganga, and the Brahmaputra
deposits of glacial clays which are extensively river system.
used in the cultivation of Zafran in the Kashmir  Statement 4 is incorrect: Western coasts are an
Himalayas. Duns are found not only in example of submergent coast and it is for this
Uttarakhand but also in Jammu and Kashmir. The reason that it has ideal conditions for the
longitudinal valleys found in the state of Jammu development of natural ports and harbors. It has
and Kashmir are called as the dunes e.g., Jammu no role in the shaping of the present day
Dun and Pathankot dun. In the state of Himalayan river system.
Uttarakhand, Dehradun is the largest of all the
duns. Q.10) Ans: a
 Statement 2 is incorrect: Bugyals are the summer Exp:
grasslands found at higher altitudes in the state of  Statement 1 is correct: Jhelum rivers originate at
Uttarakhand. While they are mostly snow covered the Verinag spring in the foothills of Pir-Panjal in
in winters, nomadic groups migrate to these the Kashmir valley. It is an important tributary of
grasslands during summers for pasture. Indus. It flows through Srinagar and also the
Wular Lake. It joins the Chenab river near Jhang,
Q.8) Ans: d Pakistan.
Exp:  tatement 2 is correct: Ravi river, a tributary of
 Statement 1 is incorrect: The drainage basin of Indus, originates near west of Rohtang pass, Kullu
Himalayan rivers is more than the peninsular hills in Himachal Pradesh. It joins Chenab at
rivers. Himalayan rivers are perennial as they are the Sarai Sindhu. It also drains the Chamba Valley
continuously fed with water from rainfall and also of Himachal Pradesh.
from the melting of the glaciers. They have a  Statement 3 is correct: Indus rivers only pass
longer course and flow through mountainous through Leh district of UT of Ladakh in India.
terrains with frequent shifting of their course.
Peninsular rivers on the other hand have a Q.11) Ans: b
relatively smaller basin. They are mostly fed from Exp:
e

monsoonal rains and hence are seasonal in  Statement 1 is incorrect: Alaknanda originates
in

nature. They are smaller rivers with fixed courses. at Satopanth glacier above Badrinath. It joins
nl

 Statement 2 is incorrect: Himalayan rivers exhibit Bhagirathi (source: Gomukh) at Dev prayag where
l.o

antecedent, consequent, dendritic (in plains) it is known as Ganga. Alaknanda is composed of


drainage patterns through their course. Hence,
ria

Dhauli and Vishnu ganga which meet at Vishnu


they do not exhibit a single pattern. Prayag or Joshimath. Hence, Joshimath is
e

associated with river Alaknanda.


at

Q.9) Ans: b  Statement 2 is correct: The total length of Ganga


cm

Exp: is 2525 Km, with the maximum part in Uttar


ps
.u

PRAYAS TEST 9 15
w
w
w
Contact us : info@onlyias.com

OnlyIAS Nothing Else Visit : dpp.onlyias.in


Contact : +91-7007 931 912

Pradesh, West Bengal, Bihar and the least in the population density of One-horned rhinoceros and
state of Uttarakhand. is referred to as the Mini Kaziranga.
 Statement 3 is incorrect: The Ganga river system  Statement 3 is incorrect: Nameri National Park is
includes Yamuna as well as its tributaries. located in Assam. It shares its boundary with the
Chambal, Sindh, Betwa, Ken, etc. are the Pakhui Wildlife Sanctuary of Arunachal Pradesh. It
tributaries of Yamuna that originate in the has been declared as a Tiger Reserve. It does not
Peninsular Plateau. Hence, they form a part of the lie to the close vicinity of Brahmaputra.
Ganga river system and are not of Himalayan
origin. Q.15) Ans: b
Exp:
Q.12) Ans: c The guidelines to be followed for declaring the onset
Exp: of monsoon over Kerala and its further advance over
 Statement 1 is correct: River Alaknanda finds its the country have been broadly laid out by the Indian
source in the Satopanth Glaciers located above Meteorological Department (IMD)
Badrinath.  Option 1 is incorrect: The first guideline relates to
 Statement 2 is correct: River Yamuna originates the amount of rainfall received.
from the Yamunotri  § If after 10th May, 60% of the available 14
glacier near Bandarpunch range. It is a tributary stations enlisted*, viz. Minicoy, Amini,
of Ganga and meets Ganga at Prayagraj, Uttar Thiruvananthapuram, Punalur, Kollam, Allapuzha,
Pradesh. Kottayam, Kochi, Thrissur, Kozhikode, Thalassery,
 Statement 3 is incorrect: The river Son is the Kannur, Kudulu and Mangalore report rainfall
largest south bank tributary of Ganga. It originates of 2.5 mm or more for two consecutive days.
at the Amarkantak Plateau and joins Ganga at  Thus, rainfall received over weather stations
Arrah, near Patna, Bihar. in Lakshadweep (Minicoy)
 Statement 4 is incorrect: The river Kosi originates and Karnataka (Mangalore) are also taken into
in Tibet, north of Mount Everest. It is joined by account, along with those with located in Kerala
rivers Son Kosi and Taimur Kosi and finally (Alappuzha, Kollam etc)
emerges as Sapt Kosi after joining river Arun.  Options 2 and 3 are corrects
● The second guideline relating to the wind
field
Q.13) Ans: b o Depth of westerlies (i.e. Atmospheric pressure
Exp: exerted by the westerly) should be maintained
 Statement 1 is incorrect: The river Brahmaputra upto 600 hPa, in the box equator to Lat. 10ºN and
originates in the Chemayungdung Glacier near Long. 55ºE to 80ºE. The zonal wind speed over the
the MAnsarovar Lake. area bounded by Lat. 5-10ºN, Long. 70-80ºE
 Statement 2 is correct: Majuli, Assam has been should be of the order of 15 – 20 Kts. at 925 hPa.
regarded as the Island district of India. Majuli is The source of data can be RSMC wind
the largest river Island of the world. analysis/satellite derived winds.
● The third guideline is related to the
e

Q.14) Ans: a Outgoing Longwave Radiation (OLR)


in

Exp: o INSAT derived OLR value should be below 200


nl

 Statement 1 is correct: Kaziranga National Park, a wm-2 in the box confined by Lat. 5-10ºN and
l.o

World Heritage Site, is located in Assam. It is Long. 70-75ºE.


ria

known for one horned rhinoceros. River


Brahmaputra flows through this national park. Q.16) Ans: d
e

 Statement 2 is correct: Pobitora Wildlife Exp:


at

sanctuary in Assam is located on the south bank


cm

of river Brahmaputra. It houses the largest


ps
.u

PRAYAS TEST 9 16
w
w
w
Contact us : info@onlyias.com

OnlyIAS Nothing Else Visit : dpp.onlyias.in


Contact : +91-7007 931 912

Exp:

 Statement 1 is incorrect: The North Eastern


Monsoon has little to zero correlation with the
In India, If rain fails to occur for one or more weeks Position of the Himalayas. They are in
during the south-west monsoon period after having fact a manifestation of the global wind belts.
rained for a few days, such a phenomenon is called  The easterlies blow naturally between 0 degrees
a Monsoon-Break. There might however be a rainfall (equator) and 30 degrees Latitude. Hence even if
increase in a narrow belt along the foothills of there were no Himalayas, the north eastern
Himalayas, and Northeast India. easterlies would still have picked up
 The month of June is an onset month where no moisture from the Bay of Bengal. The shores of
breaks are usually observed as Monsoon is in its Tamil Nadu would still get the same amount
progress state. September also being a of rainfall from the North Eastern Monsoon in
withdrawal month does not see a break period as the month of December and January.
well. It is generally July and August that see  Statement 2 is incorrect: The onset of Monsoon
break Monsoon conditions. in India takes place due to the seasonal reversal of
 To understand this phenomenon, one needs to winds. The South Western Monsoon winds
understand the meaning of a Monsoon Trough. originate from MASCARENE (near Madagascar in
 § A trough is a belt of low pressure extending to a Western Indian Ocean) and are pulled by the low
large area. This trough is seen during monsoon pressure prevailing ( due to excess heating ) over
period, hence known as Monsoon trough. This the Tibetan Highlands during the summer
trough is located in east west direction from season.
Northwest Rajasthan till the Bay of Bengal.  Thus, even if The Himalayas were absent, the
§ During break Monsoon, the trough shifts seasonal reversal would still have taken place and
closer to the foothills of Himalayas or sometimes the onset of the monsoon would occur at its
not visible at all. This takes place due to dynamic usual time (around June 1st week).
factors not very well understood.  Statement 3 is correct: One of the most
 The upper atmospheric jet Streams also play a important ways in which the Himalayas influence
role in forcing a High Pressure situation over the the Indian climate Is by trapping the monsoon
Indian plains which results in a prolonged period winds, forcing them to shed their moisture
of no rains and hence a Monsoon-Break. within the subcontinent.
 Had there been no Himalayas, the South Western
e

 Over the west coast, prolonged dry spells are


in

associated with days when winds blow parallel to monsoon winds would have had an almost
nl

the coast. uninterrupted passage over the Indian


mainland. This would leave little scope for any
l.o

Q.17) Ans: c kind of orographic rainfall over the Indian


ria

subcontinent.
 Thus the Long Period Average value of the rainfall
e
at

recorded in India would be lower than the present


value (89cm) as other mountain ranges (western
cm

Ghats, Satpuras and the Vindhyas) would still


ps

cause some, if not much, orographic rainfall.


.u

PRAYAS TEST 9 17
w
w
w
Contact us : info@onlyias.com

OnlyIAS Nothing Else Visit : dpp.onlyias.in


Contact : +91-7007 931 912

 Statement 4 is incorrect: § The SST is restricted to a few millimetres of the top


 The prime reason for the dry conditions of the ocean layer and is largely influenced by strong winds,
Thar Desert is that while the Bay of Bengal evaporation, or thick clouds
branch of the monsoon sheds most of its § The variations in the upper ocean thermal energy
moisture by the time conditions are mainly responsible for the summer
it reaches the Thar Desert, the Arabian Sea monsoon.
branch of the monsoon winds blow almost
parallel to the the Aravali ranges and hence more Q.19) Ans: c
or less bypass maximum parts of Rajasthan. Exp:
 § Himalayas therefore have little to zero role to
play here.

Q.18) Ans: d
Exp:
 All options are correct
 Sea surface temperature (SST) is routinely used
for predicting whether the total amount of
rainfall that India receives during the monsoon
season will be less or more than the long-term
mean of 887.5 mm. The information on whether  Statement 1 is incorrect: The Indian Ocean Dipole
the amount of monsoon rainfall will be more or - often called the "Indian Niño" because of its
less than the long-term mean is generally made similarity to its Pacific equivalent - refers to the
available by the beginning of April. The Earth difference in sea-surface temperatures in
System Science Organisation (ESSO) and the opposite parts i.e Western and Eastern parts of
India Meteorological Department (IMD) take into the Indian Ocean.
account five conditions while making forecasts  Statement 2 is incorrect: Both El Nino and Indian
about the monsoon in April every year. Ocean Dipoles are marked by irregular oscillation
§ The SST gradient between the north Atlantic and of sea surface temperatures. To put it simply,
the north Pacific (conditions during December of the they can only be observed and have little
previous year and January of present year) predictive value. Although there are research
§ SST of the equatorial south Indian papers available that claim to have found the
Ocean (conditions during February and March of the parameters required to predict these events,
present year) there is, in general, no consensus as such.
§ Mean Sea Level pressure in east Asia (conditions
during February and March of the present year)
§ The surface air temperature over northwest
Europe (conditions during January of the present
year)
§ The warm water volume of the equatorial Pacific
e

Ocean (conditions during February and March of the


in

present year)
nl

o In recent times however, scientists are increasingly


l.o

of the opinion that ocean mean temperature


(OMT) has a better ability to predict this than sea
ria

surface temperature. Reasons :


e

§ Sea surface temperature gives information only  Statement 3 is correct: The 2019-20 Monsoon
at

about the thin upper layer of the ocean and does not season saw floods in India and bushfires in
cm

reflect the thermal energy available in the upper Australia. From September 2019 until March
ocean 2020, when the final fire was extinguished,
ps
.u

PRAYAS TEST 9 18
w
w
w
Contact us : info@onlyias.com

OnlyIAS Nothing Else Visit : dpp.onlyias.in


Contact : +91-7007 931 912

Australia had one of the worst bushfire seasons in Indian Ocean and which generally yields normal
its recorded history. At the same time, about a or excessive rainfall the following season.
million people were displaced in Karnataka and  The intraseasonal variability of rainfall during a
Maharashtra due to ravaging floods in India. monsoon season is characterized by
 Scientists and Meteorologists around the world the occurrence of active and break phases.
have concluded that one of the prime reasons for  § During the active phase, the rainfall is above
such simultaneous and contrasting events was the normal over central India and below normal over
occurrence of a positive phase of Indian Ocean northern India (foothills of the Himalaya) and
Dipole. southern India. This pattern is reversed during the
 With a warmer than usual Arabian Sea, there break phase (also called Monsoon Breaks).
was more moisture available for the South
Western monsoon winds which eventually
Wreaked havoc in the states of Karnataka,
Maharashtra and Kerala.
 § At the same time, a cooler than average
Eastern Indian Ocean ( as shown in the map
above) induced aridity, dryness and anticyclonic
high pressure conditions in Northern
Australia. Although bushfires are common in
Australia, however the positive IOD drastically
increased the intensity of the bushfires.

Q.20) Ans: a
Exp:  Option 3 is not correct:
 Western Disturbances are
phenomena independent of the phenomenon/
mechanism of Monsoon season in India.
A western disturbance is an extratropical storm
originating in the Mediterranean region that
brings sudden winter rain to the northern
parts of the Indian subcontinent. It is a non-
monsoonal precipitation pattern driven by
the westerlies.
 Option 5 is not correct: The withdrawal of
monsoon in India is a gradual process, Unlike the
onset of the monsoonal winds. The North Eastern
trade winds gradually push the southwestern
westerlies as the Intertropical convergence zone
 Options 1, 2 and 4 are correct: (ITCZ) starts moving southwards in September.
e
in

 The Monsoonal winds typically burst onto


Q.21) Ans: b
nl

the shores of Kerala in the 1st week of JUNE. This


Exp:
meAns that at the time of its arrival, the
l.o

 El Nino is a phenomenon in the equatorial Pacific,


normal rainfall increases suddenly and continues
ria

in which sea-surface temperatures rise over a


constantly for several days.
threshold of +0.5 degree Celsius. During El Nino
 Indian Summer Monsoon Rainfall exhibits a
e

years, the warmth shifts to Central and East


at

prominent inter-annual variability. A season


Tropical Pacific and along with it cloudiness and
of deficient June to September monsoon
cm

rainfall (near western coast of PERU)


rainfall in India is followed by warm sea surface
ps

temperature (SST) anomalies over the tropical


.u

PRAYAS TEST 9 19
w
w
w
Contact us : info@onlyias.com

OnlyIAS Nothing Else Visit : dpp.onlyias.in


Contact : +91-7007 931 912

 La Nina is a phenomenon which has the opposite  They are acidic mainly due to the nature of the
effect of El Nino. The sea surface temperature parent rocks. The alkali content is fair. They
across the eastern equatorial part of the Pacific are poor in lime, magnesia, phosphates, nitrogen
Ocean will be lower than normal by 3-5 °C. and humus. They are fairly rich in potash and
 Statements 1 and 2 are correct: potassium.
 Although both El-nino and La-Nina (together  They occupy about 3.5 lakh sq km (10.6 per cent)
called the El Nino Southern Oscillations or of the total area of the country. These soils are
ENSO) are phenomena which take place in spread on almost the whole of Tamil Nadu.
the Pacific Ocean, however both the Other regions with red soil include parts of
processes can have a significant say in India's Karnataka, south-east of Maharashtra, Telangana,
monsoon and hence economy. In meteorological Andhra Pradesh, Madhya Pradesh, Chhattisgarh,
parlance, these interconnected events are called Odisha etc.
“TELECONNECTIONS.”  The red soils, with the proper use of fertilizers and
 § The El-Nino, generally has a negative impact on irrigation techniques, give good yield of cotton,
the Indian monsoon i.e. it suppresses the wheat, rice, pulses, millets, tobacco, oil seeds,
monsoonal rainfall. This invariably is associated potatoes and fruits.
with droughts in many parts of the country which  The red colour is due to the presence of iron
has a ripple effect on various economic indicators oxide. In oxidizing conditions, rust or iron oxide
such as inflation and prices of commodities. develops in the clay, when the soil is
§ While La Niña conditions enhance the rainfall present above the water table giving the soil
associated with the Southwest monsoon, it has a characteristic red colour. The colour is more
a negative impact on rainfall associated with the due to the wide diffusion rather than high
Northeast monsoon. percentage of iron oxide content.
§ During La Niña years, low pressure or cyclones
formed in the Bay of Bengal remain significantly Q.23) Ans: c
to the north of their normal position. Exp:
§ Besides, instead of moving westwards, these  Semul tree (Bombax Ceiba)
systems recurve. As they lie to the north of their  It is found in the Moist Deciduous Forests which
normal position, not much rainfall occurs over record rainfall between 100 to 200 cm.
southern regions like Tamil Nadu.  Also called Malabar silk-cotton tree; red silk-
 Statement 3 is not correct cotton, its flowers spectacularly in spring .
 When there is high snowfall in the Tibetan plateau  From the Bhil tribe in Rajasthan to The Khuman
during the winter season, it is generally seen that clan of the Meetie community in Manipur, many
the following monsoon in india is considerably tribal communities consume semal because of its
weak. The tropical easterly jet stream plays an medicinal properties while others worship and
important role in the manifestation of this protect the tree.
phenomenon. To put it simply, when there is a  § Ecologically, the tree plays an important role in
thick snow cover on the Tibetan plateau , it attracting several creatures like birds, butterflies,
prevents the development of strong low bees, and spiders.
e

pressure conditions (prerequisite for driving the  The Simlipal National park derives its name from
in

South Western monsoon winds). this tree. It is naturally found all over India
nl

especially along the foothills of the Himalayas,


l.o

Q.22) Ans: a the moist areas of the peninsula and the states
Exp:
ria

of Odisha and West Bengal.


 Statements 1 and 2 are correct
e

 CHIEF CHARACTERISTICS OF RED SOILS: Q.24) Ans: d


at

 The main parent rocks are crystalline Exp:


cm

and metamorphic rocks like acid granites,


gneisses and quartzites.
ps
.u

PRAYAS TEST 9 20
w
w
w
Contact us : info@onlyias.com

OnlyIAS Nothing Else Visit : dpp.onlyias.in


Contact : +91-7007 931 912

contain sodium, but it should only be 0.2 percent


sodium or less. More sodium can change
the salinity of your soil, which can kill many
plants.
 Some plants, especially vegetables that have lots
of seeds inside, such as tomatoes (Lycopersicon
esculentum), need extra calcium as they grow,
and the dolomite is an excellent way to provide
that nutrient. Dolomite also helps provide
magnesium and other nutrients that are beneficial
to growing plants. Magnesium excels at
 All Options are correct neutralizing acid when the garden soil is too acidic
 Thorny Scrub Forests of India for the plants that will grow in it.
§ These are forests which receive rainfall less
than 50 cm annually. Q.26) Ans: b
§ These consist of open woodland with thorny Exp:
trees with short trunks and low, branching
crowns; spiny and xerophytic shrubs; and dry
grassland.
§ The trees are low and widely scattered. Acacias
and Euphorbias are very prominent along with
Khairm babool, neem and palas etc.
§ This is the habitat of the great Indian bustard
and blackbuck,
§ The remaining natural habitat is threatened by
overgrazing and invasive weeds
§ TRajasthan, south-western Punjab,
western Haryana, Kutch and neighbouring parts
of Saurashtra (gujrat) .
§ Such forests also grow on the leeside of the
Western Ghats covering large areas of
 Ebony and Mahogany are examples of hardwood
Maharashtra (Vidarbha), Karnataka (Hyderabad-
trees found in Evergreen Forests
Karnataka), Telangana, Andhra Pradesh and Tamil
 Options 1 and 3 are correct:
Nadu.
 Dandeli Anshi (karnataka, Western ghats)
Q.25) Ans: a and Cambell bay (Great nicobar) contain
Exp: evergreen vegetation.
 Dolomite, a type of limestone, provides valuable  Options 2 and 4 are incorrect
nutrients to plants and helps change the pH of  Mukundara hills (Rajsthan) and Gugamal MP
e

the soil by raising it to match the plants' needs. (Maharasthra) fall under the dry deciduous
in

It's sometimes called dolomitic lime or dolomitic vegetation category in india.



nl

limestone, and provides more nutrients than While Gugamal NP receives moderate rainfall due
l.o

straight line. to its location in the leeward side of the Western


 Dolomite powder is a limestone compound that Ghats, the Mukundara hills NP receive
ria

contains calcium and magnesium. The best medium rainfall due to its interior location in the
Central highlands of India.
e

mixtures contain 8 to 12 percent magnesium and


at

18 to 22 percent calcium. These elements


Q.27) Ans: b
cm

help influence the pH of the soil and provide


valuable nutrients to plants. Dolomite can often Exp:
ps
.u

PRAYAS TEST 9 21
w
w
w
Contact us : info@onlyias.com

OnlyIAS Nothing Else Visit : dpp.onlyias.in


Contact : +91-7007 931 912

 Statement 2 is correct: As can be seen in the


table, Total area covered by moist deciduous
forests is almost 25 times the area covered
by Littoral and Swamp Forests.

 Shelterbelts
 In general, shelterbelts (windbreaks) are strips of
vegetation composed of trees and shrubs grown  Statement 3 is correct: These forests exist mostly
along the coasts to protect coastal areas from in the eastern part of the country – northeastern
high velocity winds and also from devastations states, along the foothills of the Himalayas,
like the ones caused by tsunami. Jharkhand, Andhra Pradesh, West Orissa and
Chhattisgarh, and on the eastern slopes of the
Western Ghats.
 The Eastern slopes of the Southern
W.Ghats receive rainfall from both the South
West monsoon and NorthEast Monsoon.
Similarly, Parts of Coastal Andhra Pradesh are also
influenced by rains during both the seasons.
 The interesting thing here is despite getting rains
Example: SunderbAns act as natural shelterbelts in during both the Southwest monsoon and the
West Bengal and help minimize the impact of various Northeast monsoon, the amount of rain received
cyclones in the area. is not enough (for them to be categorized as
o They also enhance the biodiversity in the area Evergreen Forests) and hence tree leaves are
while helping enhance soil fertility and health too. invariably shed during some point in the spring
Agriculturists have a long history of enhancing crop season.
growth by manipulating soil and plant
microclimates, through use of irrigation,
glasshouses, shelterbelts and windbreaks, snow Q.29) Ans: d
fences, wind machines, surface mulches, certain Exp:
tillage practices, alley cropping, and agroforestry
o They also serve the purpose of sand binders and
prevent sand erosion.

Q.28) Ans: c
e

Exp:
in

 Statement 1 is incorrect: The moist deciduous


nl

forests are found in the regions, which record


l.o

rainfall between 100 and 200 cm. The trees drop


their leaves during the spring and early
ria

summer when sufficient moisture is not available


e

(Autumn is the season between summers and


at

winters). This process is known as Abscission.


cm

They are also called ‘Monsoon forests.’


ps
.u

PRAYAS TEST 9 22
w
w
w
Contact us : info@onlyias.com

OnlyIAS Nothing Else Visit : dpp.onlyias.in


Contact : +91-7007 931 912

that contain iron are metallic


known as ferrous minerals that
minerals. These don't contain
minerals are iron are known
magnetic and less as non-ferrous
resistant to corrosion minerals. They
than non-ferrous don't have
minerals. magnetic
Chromite, manganese, properties and
Pyrites, Tungsten, Nickel, are generally
Cobalt more resistant
to corrosion
than ferrous
minerals.
Gold, Silver, Copper,
Lead, bauxite,
Magnesium,Tin
 All options are correct: o Nonmetallic minerals lack metallic characteristics
 Natural gas (also called fossil gas; sometimes just like good electric and thermal conductivity, luster,
gas) is a naturally occurring hydrocarbon gas rigor, and malleability; they are, however, essential
mixture consisting primarily of methane, but for many industries.
commonly including varying amounts of § Examples: Limestone,nitrate,Potash,mica,
other higher alkanes, and sometimes a small gypsum, Dolomite.
percentage of carbon dioxide, nitrogen, hydrogen
sulfide, or helium. Q.31) Ans: c
§ It is formed when layers of decomposing plant and Exp:
animal matter are exposed to intense heat and Iron ore: Iron is taken out from mines in the form of
pressure under the surface of the Earth over millions iron ore. There are different types of iron ore
of years. containing varying percentage of pure iron:
§ Natural gas is a non-renewable hydrocarbon fossil  Haematite
fuel, found in deep underground rock formations or § 60 to 70 percent pure iron
associated with other hydrocarbon reservoirs in coal Odisha was the leading producer of iron ore
beds and as methane clathrates. accounting for 59.64% of total production
§ As can be seen in the table, the PSUs have detected followed by Chhattisgarh (14.11%), Karnataka
recoverable natural gas reserves in various states. (12.76%), Jharkhand (10.93%). (Source annual
report 2020-21 of Min of mining).
Q.30) Ans: b § Formed in the Cuddapah rocks and Dharwad
Exp: System
 Options 1,2,3 and 5 are correct:
e

 Magnetite
in

 Metallic minerals are the minerals containing one § magnetite actually has higher iron content (70-
nl

or more metals. These usually occur as mineral 75%) than the mineral hematite. However, while
l.o

deposits and are a great heat and electricity hematite ore generally contains large
conductor. They can be ferrous or non-ferrous. concentrations of hematite, magnetite ore
ria

Ferrous Non Ferrous generally holds low concentrations of magnetite.


e

§ Hence the ore needs to be beneficiated (treat


at

 Ferrous minerals -  Non-Ferrous to improve its properties) for magnetite recovery.


cm

The metallic minerals minerals: The § Taconite and Lodestone are forms of Magnetite
ps
.u

PRAYAS TEST 9 23
w
w
w
Contact us : info@onlyias.com

OnlyIAS Nothing Else Visit : dpp.onlyias.in


Contact : +91-7007 931 912

§ Distribution: Odisha, Jharkhand, Chhattisgarh,


Andhra Pradesh,
 Limonite
§ inferior iron ore that is yellowish in colour with
40 to 60 per cent iron content
§ Distribution : Raniganj, Garhwal (uttarakhand)
and Kangra Valley (himachal)
 Siderite
§ It is iron carbonate ore of inferior quality
with Iron content in siderite – 40-50% per cent .
§ Invariably has many impurities and hence
mining, in many places, is economically unviable. ● Mines in ODISHA
§ Self-Fluxing due to natural presence of o As can be seen in the map, Odisha has large
Limestone (Separates impurities on its own, when reserves of almost all kinds of minerals that are
in liquid form, without addition of extra flux commercially exploited for various industries in India.
agents) o Odisha has the largest reserves of bauxite, iron
ore and Chromite in India.
Extra Edge by ONLYIAS:  Odisha with 70.95% contribution was the
● Iron ore details: leading producer of bauxite followed by
 India has large iron ore reserves. It occurs in Gujarat (9.50%), Chhattisgarh (7.18%),
various geological formations but major Jharkhand (6.50%), Madhya Pradesh (3.14%),
economic deposits are found in volcano- and Maharashtra (2.73%).
sedimentary Banded Iron Formation (BIF) from  Odisha reported almost entire production of
the Precambrian age. chromite.
 India's leading state that produces iron ore is
Odisha. It accounts for more than 55% of the
total production followed by Chhattisgarh Q.33) Ans: a
producing almost 17%.This is followed by Exp:
Karnataka and Jharkhand producing 14% and  Statement 1 is correct:
11% respectively.  Diamond occurrences have been reported since
 In India, the value of metallic minerals in 2018- prehistoric times in the country.
19 was at INR 64,044 crores. Among these the Presently, diamond fields of India are grouped
principal metallic minerals, iron ore contributed into four regions:
more than 45000 crore rupees. § South Indian tract of Andhra Pradesh,
comprising parts of Anantapur, Kadapa, Guntur,
Krishna, Mahabubnagar and Kurnool districts;
Q.32) Ans: d § Central Indian tract of Madhya Pradesh,
Exp: comprising Panna belt;
§ Behradin-Kodavali area in Raipur district and
e
in

Tokapal, Dugapal, etc. areas in Bastar district


of Chhattisgarh;
nl

§ Eastern Indian tract mostly of Odisha, lying


l.o

between Mahanadi and Godavari valleys.


ria

§ By States, Madhya Pradesh accounts for about


90.18% resources followed by Andhra Pradesh
e

5.72% and Chhattisgarh 4.09%


at

 Statements 2 and 3 are incorrect


cm

 India depends largely on imports of rough gem


diamonds for its Cutting and Polishing Industry as
ps
.u

PRAYAS TEST 9 24
w
w
w
Contact us : info@onlyias.com

OnlyIAS Nothing Else Visit : dpp.onlyias.in


Contact : +91-7007 931 912

there is no notable production except for two


producers in Madhya Pradesh whose limited
production is too sparse to meet the Cutting and
Polishing Industry's requirements. The cut and
polished diamonds are predominantly re-
exported.

 § These placer deposits, also called Alluvial


gold is obtained from the Sands of
the subarnarekha river and the Sona nadi in
the Singhbhum District of Jharkhand
§ In India, there are two reporting mines, both
under Public Sector located in Panna district of
Madhya Pradesh.  Statement 3 is Correct:
 Maikal Range, mountain range in Madhya
Q.34) Ans: d Pradesh state, central India. It runs in a north-
Exp: south direction and forms the eastern base of the
 Statement 1 is Correct: Limestone often triangular Satpura Range. The Maikala Range
contains magnesium carbonate, either as consists of laterite-capped, flat-topped plateaus.
dolomite CaMg (CO3 ) 2 or magnesite (MgCO3  § Mineral deposits found here include coal,
) mixed with calcite. Such rocks are termed limestone, bauxite, corundum, dolomite, marble,
as 'dolomitic' or 'magnesian' limestone. slate, and sandstone.
Limestone altered by dynamic or contact
metamorphism become coarsely crystalline and Q.35) Ans: b
are referred to as 'marble' and 'crystalline Exp:
limestone'
 Although the largest
reserves are located in Andhra Pradesh, bu
Rajasthan was
the leading producing State accounting for (20%)
of the total production of limestone, followed
by Madhya Pradesh & Andhra
Pradesh (13% each)
 Kurnool, Krishna and Cuddapah districts
e

of Andhra Pradesh are important mines.


in

 Statement 2 is Correct:
nl

 Placer deposit or placer is an accumulation of


l.o

valuable minerals formed by gravity Statement 1 is incorrect and Statement 2 is correct:


separation from a specific source
ria

Rare Earths are a group of 17 elements starting with


rock during sedimentary processes. Gold is often lanthanum in the periodic table of elements and
e

liberated from the Rocks by weathering and its include scandium and yttrium. They
at

particles get concentrated at certain places in the are moderately abundant in earth's crust but not
cm

rivers. Such deposits are then recovered by a concentrated enough to make them economically
process called panning exploitable. A number of rare-earth minerals
ps
.u

PRAYAS TEST 9 25
w
w
w
Contact us : info@onlyias.com

OnlyIAS Nothing Else Visit : dpp.onlyias.in


Contact : +91-7007 931 912

contain thorium and uranium in variable amounts,  Statement 1 is incorrect: Western Disturbances
but they do not constitute essential components in move Eastward towards northwest India and
the composition of the minerals.The principal sources Pakistan under the influence of Subtropical
of Rare Earths are bastnaesite, xenotime, Westerly Jet Stream and Westerlies.
and monazite (a phosphate).  Statement 2 is correct: The moisture in Western
 Monazite is radioactive (presence Disturbances usually comes from Mediterranean
of Thorium and Uranium) is often found in placer Sea and Atlantic Ocean. On their way they pick up
deposits. India, Madagascar, and South moisture from Caspian Sea and Black Sea as well.
Africa have large deposits of monazite sands. The  Statement 3 is correct: High pressure over
deposits in India are particularly rich in monazite Ukraine and neighbourhood areas causes the
and are found in sands of both rivers and intrusion of cold air from the Polar region towards
beaches. Mediterranean Sea. This generates favourable
conditions for cyclogenesis in the upper
atmosphere, which promotes the formation of an
eastward-moving extra tropical depression.

Q.38) Ans: c
Exp:
 Option A is incorrect: Omega block is a high-
pressure pattern that blocks and diverts jet
streams.
 Option B is incorrect: When an omega shaped
wave is present on the jet stream which arcs over
Europe, warm dry air from southern Europe and
Africa can be pulled north, pushing temperatures
 Statement 3 is correct and 4 is incorrect:
higher than normal.
o As can be seen in the table, Andhra pradesh,
Tamil Nadu and Odisha have appreciable  Option C is correct: An important consequence of
monazite resources. this phenomena is formation of Spanish plume, in
o Uranium and Thorium for India’s nuclear which very warm air moving northwards from
program are sourced from these regions. Spain towards the UK rises and leads to formation
of cumulonimbus clouds and heavy rainfall.
Q.36) Ans: c
Exp:
 Statement 1 is correct: The eye of a tropical
cyclone is a region of calm and subsiding warm
air. There is little or no precipitation and
sometimes blue sky or stars can be seen.
 Statement 2 is correct: Ordinary thunderstorms
e

develop vertically, meaning upright from the


in

ground. In the tropical cyclones, there is high


nl

rotational energy and air does not climb up


directly. Instead, it takes a swirly, roundabout
l.o

path. So, while they reach the height of 10 to 12


ria

kilometers the rising motion isn’t quite as strong


to produce lightning.
e
at

Q.37) Ans: c
cm

Exp:
ps
.u

PRAYAS TEST 9 26
w
w
w
Contact us : info@onlyias.com

OnlyIAS Nothing Else Visit : dpp.onlyias.in


Contact : +91-7007 931 912

 Statement 3 is correct: During El-Nino year sea


surface temperature of western Indian Ocean
increases and this leads to bleaching of corals.
Such large scale bleaching was seen in 1998 and
recently as well.

Q.40) Ans: b
Exp:
 Statement 1 is incorrect: The walker circulation is
a Convective cell of air circulation which is formed
due to the development of a pressure gradient
from East to West in the equatorial Pacific Ocean.
 Statement 2 is correct: Walker circulation
facilitates Upwelling of cold water near Peru coast
resulting in High Pressure atmospheric stability
and dry weather. Also upwelling of nutrients from
below the ocean facilitates fishing in this region,
which is beneficial for the people of Peru.
Otherwise, during the El-Nino period, due to
break in upwelling of water, fishes die due to
 Statement 3 is correct: La-Nina is an opposite
phenomenon of El-Nino and is a strengthened
phase of walker circulation.

Q.41) Ans: a
Exp:
Q.39) Ans: d  Statement 1 is correct: The Savanna are the
Exp: tropical grasslands characterized by a distinct wet
 Statement 1 is correct: El-Nino results in floods in and dry season. They are most developed in
Peru while Pacific coast of Australia and Indonesia Sudan and hence also called the Sudan climate.
have abnormally dry conditions that forces the They are found in West African Sudan, Eastern
collapse of Agriculture with instances of Forest Africa, and the southern parts of Africa above the
fire. tropic of Capricorn. There are two distinct regions
 Statement 2 is correct: During an El-Nino year, of Savanna both North and South of the equator
fewer hurricanes form over the Atlantic Ocean, in South America. Llanos are found near Orinoco
the Caribbean Sea and the Gulf of Mexico due to basin whereas Campos are found in the Brazilian
increased wind shear. The increased wind shear highlands.
helps to prevent tropical disturbances from  Statement 2 is incorrect: The dominant winds in
developing into hurricanes. the Savanna regions are the Trade winds which
e

bring rainfall to the coastal districts of these


in

areas.
nl
l.o

Q.42) Ans: d
Exp:
ria

 Statement 1 is correct: Sunspot cycles play an


e

important role in shaping the climate of the Earth.


at

They appear as dark and cooler patches on the


cm

surface of the Earth. The increase in sunspots on


the sun is associated with cooler and wetter
ps
.u

PRAYAS TEST 9 27
w
w
w
Contact us : info@onlyias.com

OnlyIAS Nothing Else Visit : dpp.onlyias.in


Contact : +91-7007 931 912

climatic conditions while their decrease is rain shadow area formed by being on the leeward
associated with warmer and drier conditions of side of the Andes. Hence, all the deserts in South
climate on earth. America are not formed due to continentality.
 Statement 2 is correct: Volcanic eruptions are  Statement 2 is correct: Western boundaries of
responsible for the release of excessive hot gases, the African deserts like the Sahara and the Namib
and aerosols on the earth’s atmosphere. Increase Deserts is flanked by the cold water
in such eruptions causes greater accumulation of currents. Canaries current near the Sahara desert
these materials resulting in warming of the Earth. and the Benguela current near the Namib flank
 Statement 3 is incorrect: Aerosol Radiative the boundaries of these deserts.
Forcing is a method of quantifying the effect that  Statement 3 is correct: Hot deserts are the
aerosol produces on the climate. It is found that hottest regions of the world. They are mostly
excessive emission of aerosol by human activities found on the western coasts of the continents
is responsible for changing the earth's climate. between 15 to 30 degrees North and South of the
Hence, it is not a natural rather an anthropogenic equator. They lie near the Horse latitudes or the
factor. Sub-tropical high-pressure belts which
 Statement 4 is correct: Milankovitch oscillations experience descending air and negligible
are associated with the changes in the axial tilt, precipitation. The diurnal range of temperature is
revolution of the earth around the sun, extremely high in the deserts.
eccentricity. These factors together are
responsible for changing the amount of insolation Q.45) Ans: d
received on Earth and hence influence the climate Exp:
of the Earth.  Statement 1 is incorrect: The Mediterranean
 Statement 5 is incorrect: The sediments found in regions are characterized by the warm, dry
the glacial lakes are not directly responsible for summers and cool, moist winters. The rainfall in
the change of Earth’s climate rather they give the Mediterranean regions occurs in winter
information regarding the alternate dry and wet months. Despite the rainfall, the growth is slow
climates experienced by the Earth. during the winters. The climate of the
Mediterranean regions supports the growth of a
Q.43) Ans: b wide variety of crops.
Exp:  Statement 2 is incorrect: Balearic
 Statement 1 is incorrect: Agriculture sector is Islands belonging to Spain are located above
largely responsible for the emission of greenhouse Algeria in the Mediterranean Sea. They are
gases. The highest percentage of greenhouse located in the Western part of the Mediterranean
gases from agriculture is added by the Livestock Sea and not at the Center.
products. Rice accounts for the second largest
share in the emission of Greenhouse gases. Q.46) Ans: d
 Statement 2 is correct: Agriculture is the prime Exp:
occupation of the people living in the areas of  Statement 1 is incorrect: Temperate grasslands
Monsoonal climates. Crops like rice, sugarcane, are called Steppes in Eurasia. They are also found
e

jute, etc. are grown in these areas. in North and South America. In North America,
in

they are called as Prairies while in South America


nl

Q.44) Ans: b they are called as Pampas. These grasslands are


l.o

Exp: mostly found in the interior of the continents and


 Statement 1 is incorrect: The driest desert in are away from the maritime influence of the sea.
ria

South America is the Atacama Desert or the However, Pampas of South America in Argentina
e

Peruvian desert located in Chile and Peru and and Uruguay experience the maritime influence of
at

flanked by the cold Peruvian current. the sea.


cm

The Patagonian Desert is located in Argentina,  Statement 2 is incorrect: The steppes in the
Chile, and the Falkland Islands and is due to the Northern Hemisphere are mainly under the
ps
.u

PRAYAS TEST 9 28
w
w
w
Contact us : info@onlyias.com

OnlyIAS Nothing Else Visit : dpp.onlyias.in


Contact : +91-7007 931 912

influence of continentality. The summers are very are cool and dry with maritime influence. There is
warm while the winters are very cold due to this. evenly distributed rainfall throughout the year.
However, in the Southern Hemisphere, the  Statement 2 is incorrect: The Gulf climate is found
winters are mild. along the temperate eastern margins in South-
east USA along the borders of the Gulf of Mexico
Q.47) Ans: a and hence is also called as the Gulf Climate.
Exp:  Statement 3 is incorrect: The grasslands of
 Statement 1 is correct: Nanda Devi Biosphere Steppes are characterized by the absence of trees
Reserve in the state of Uttarakhand is a World with shorter grasses whereas the grasslands of
Heritage Site under the UNESCO’s Man and Savanna are characterized by tall grasses with
Biosphere Programme. Its core areas are Nanda short trees.
Devi and the Valley of Flowers National Park.
 Statement 2 is correct: Glacial Lake Outburst Q.50) Ans: d
floods are caused due to the sudden release of Exp:
water from the accumulated glacial lakes  Statement 1 is incorrect: The permanent snow
triggered by an avalanche, earthquake, landslides, cover in the Polar or the Arctic climate is
or any other anthropogenic reasons. Glacial restricted to the higher reaches of Greenland. The
retreats are responsible for the creation of an lowlands on the other hand have few months
impression on the earth’s surface which gets filled which are ice free and are characterized by
with debris and water forming a lake. This lake on Tundra. These regions are located in Alaska,
crossing its threshold causes flooding. Hence, coastal areas of Greenland, Northern Canada, and
glacial retreats are an important cause of Glacial the Arctic areas of Eurasia.
Lake Outburst Floods.  Statement 2 is incorrect: MOSAiC or the
 Statement 3 is incorrect: National Mission for Multidisciplinary Drifting Observatory for the
Sustaining Himalayan Ecosystem is a sub mission Study of Arctic Climate (MOSAiC) led by
under the National Action Plan on Climate the Alfred Wegener Institute in Germany and
Change. It is under the Ministry of Science and designed by the International Arctic Science
Technology. Committee (IASC). It is a yearlong expedition in
the Central Arctic region that aims to study the
Q.48) Ans: b climate of the Arctic.
Exp:
 Statement 1 is incorrect: Llanos are the tropical Q.51) Ans: c
grasslands in South America on the Orinoco basin. Exp:
It is associated with Columbia and Venezuela.  Statement 1 is incorrect: Narrowness of the
 Statement 2 is correct: Pampas are the temperate continents in the Southern Hemisphere are
grasslands of South America associated responsible for reducing the severity of the
with Argentina and Uruguay. winters due to the maritime influence. Regions
 Statement 3 is correct: Pustaz are the temperate with Siberian climates are present in the Cool
grasslands situated in Hungary. Temperate Continental areas and are absent in
e

 Statement 4 is incorrect: Canterbury grasslands the Southern Hemisphere. Coniferous trees can
in

are the temperate grasslands in New Zealand. withstand such cold temperatures and are hence
nl

found in the Siberian regions. However, in the


l.o

Q.49) Ans: a Southern Hemisphere such trees are not totally


absent but can be seen in the uplands of South
ria

Exp:
 Statement 1 is correct: The Warm Temperate Chile, New Zealand, Tasmania, southeast
e

Eastern Margins are also called the China type of Australia.


at

climate. It is found on the Eastern margins of the  Statement 2 is incorrect: Pine, Fir, Spruce are the
cm

continents in the temperate regions. The common coniferous forests found in the Siberian
summers are moist and warm, and the winters
ps
.u

PRAYAS TEST 9 29
w
w
w
Contact us : info@onlyias.com

OnlyIAS Nothing Else Visit : dpp.onlyias.in


Contact : +91-7007 931 912

areas. Tundra regions comprise the lowest form of Earth. Incoming solar radiation is shortwave
vegetation in the form of lichens, mosses, etc. radiation while the long wave radiations are
 Statement 3 is correct: Laurentian climates are emitted by the Earth’s surface. Greenhouse gases
the cool temperate eastern margins of the like carbon dioxide, water vapor, methane, etc.
continents in the Northern Hemisphere. They are absorb these short-wave radiations and cause the
found only in certain areas of North America and heating of the Earth’s surface leading to Global
Asia (near China, Korea, Siberia, north Japan). warming.
They are absent totally in the southern  Statement 2 is correct: Kyoto Protocol
hemisphere. (2005) and Paris agreement (CoP 21,
2015) impose legally binding obligations on the
Q.52) Ans: d member states. They are a part
Exp: of UNFCCC (United Nations Framework
 Statement 1 is correct: The African Savanna Convention on Climate change) that was signed in
regions are a home to a number of wild animals, Rio at Earth’s Summit in 1992.
which are trapped and hunted by tourists every  Statement 3 is incorrect: Indian NDC (National
year. Hence, it is also called the land of big games. Determined Contributions) include:
 Statement 2 is correct: Prairies are known as the o A reduction in emission intensity by 33-35% of
granaries of the world as these grasslands provide GDP from 2005 levels.
for the cultivation of wheat, maize, etc. and follow o 40% share of non-fossil fuel based sources in the
extensive mechanized cultivation. share of electricity by 2030.
 Statement 3 is correct: The Siberian climate is o Additional carbon sink of 2.5 to 3 billion tonnes of
experienced in the Northern Hemisphere and is Carbon dioxide.
absent in the Southern Hemisphere. They are
defined by long cold winters and short cool Q.55) Ans: a
summers. They are also called the cold pole of the Exp:
world.  Statement 1 is correct: Neem coating of the urea
helps in reducing the emissions of nitrous oxide.
Q.53) Ans: b  Statement 2 is correct: Soil Health card monitors
Exp: the health of the soil and determines the
 Statement 1 is incorrect: Arctic Council is optimum amount of fertilizers required by the
an intergovernmental organization that works to soil. It prevents excessive use and hence controls
promote Environmental protection and the emission of nitrous oxide from the overuse of
Sustainable development of the Arctic region. It nitrogenous fertilizers.
has 8 member states: USA, Canada, Denmark,  Statement 3 is incorrect: The most dominant
Russia, Iceland, Norway, Sweden, and Finland. cause for nitrous oxide emission is the agricultural
India was given an observer status in the year production which includes the use of nitrogenous
2013. fertilizers. Even organic fertilizers can contribute
 Statement 2 is correct: India’s research to emissions due to the use of manure obtained
base, Himadri, for the study of the Arctic is by livestock.
e

deployed in Norway. Besides this India also has an


in

underwater observatory, IndARC for studying the Q.56) Ans: c


nl

Arctic climate and its linkages with the monsoon. Exp:


l.o

 Statement C is correct: The above description is


Q.54) Ans: a of British type of climate. It is also referred to as
ria

Exp: the Cool Temperate Western Margins and is


e

 Statement 1 is icorrect: Greenhouse gases are experienced in Britain and the low lying areas of
at

important in contributing to global warming. Earth north west Europe. The mean annual temperature
cm

is heated by the insolation from the sun and from ranges between 5 to 15 degree C and seldom rises
the radiation emitted from the surface of the beyond 18 degrees. It is characterized by high
ps
.u

PRAYAS TEST 9 30
w
w
w
Contact us : info@onlyias.com

OnlyIAS Nothing Else Visit : dpp.onlyias.in


Contact : +91-7007 931 912

rainfall throughout the year owing to the cyclonic Q.59) Ans: b


activity with the peak rainfall occurring during Exp:
winter or autumn months. These areas are under  statement 1 is incorrect: The Indian Standard
the influence of westerlies and have warm Meridian (ISD) passes through the states of Uttar
summers. The dominant trees are the deciduous Pradesh, Madhya Pradesh, Chhattisgarh,
trees which shed their leaves in the winter to Odisha, and Andhra Pradesh. It passes an area in
avoid frost and snow. They are the industrialized the eastern Andhra Pradesh.
and advanced areas of the world.  Statement 2 is correct: ISD covers least distance
in the state of Madhya Pradesh. It traverses
Q.57) Ans: c through its Eastern Boundary.
Exp:
 Statement 1 is correct: Steppes are the tropical Q.60) Ans: d
grasslands found in the temperate continental Exp:
areas. They are present in the areas around the  Statement 1 is correct: Mizoram is also known as
Black sea and the Caspian Sea. They are called the Molassis basin made of soft unconsolidated
Steppes in Eurasia, Pustaz in Hungary, Prairies in deposits.
North America, Downs in Australia, Velds in Africa,  Statement 2 is incorrect: Mahabharat range is
and Pampas in South America. They are located in Nepal above the Shivalik range. Hence,
characterized by short grass and absence of trees. it does not form a boundary between India and
They are referred to as the Granaries of the Nepal.
World.  Statement 3 is incorrect: Aksai Chin is an Indian
 Statement 2 is correct: Thermal ExpAnsion of the territory under the occupation of China, located in
water is one of the main reasons for the rise in the UT of Ladakh. It is a part of Ladakh
sea level as warmer water would occupy greater Plateau.
space. Other factors are: melting glaciers, changes  Statement 4 is correct: Nubra valley is a triangular
in groundwater level, etc. valley formed with the confluence of river Nubra
and Shyok. It can be accessed through the
Q.58) Ans: d Khardung La pass, which is located in Leh region
Exp: of Ladakh.
 Statement 1 is correct: Lake Priyadarshini is a
freshwater lake built by India around its research Q.61) Ans: d
station in Antarctica, Maitri. Exp:
 Statement 2 is correct: Polar Vortex is the cool  Statement 1 is incorrect: The Lipulekh pass is
low pressure area formed in the lower located at the trijunction of India-Tibet and Nepal.
stratosphere. It is weak during summers but Lampiya dura is also located in the state of
strongest during winters. At very low Uttarakhand but northern to the Lipulekh.
temperatures during winters, the lower  Statement 2 is incorrect: Mana pass is in the
temperature of the vortex leads to the formation Indian state of Uttarakhand in the Nandadevi
of Polar Stratospheric clouds. These clouds act as Biosphere reserve. Khangchendzonga Biosphere
e

a substrate for the ozone depleting substances. Reserve is a mixed Heritage site under the Man
in

 Statement 3 is correct: Vienna Convention for the and Biosphere Programme located in the state of
nl

protection of the ozone layer was signed in 1985. Sikkim.


l.o

The Montreal Protocol to the Vienna convention  Statement 3 is incorrect: Palghat pass is located in
was signed in 1987. Both are the first and the only
ria

Kerala. It is located between the Nilgiris in the


global treaty to achieve universal ratification. north and the Annamalai hills to the south.
e

The Kigali amendment to the Montreal Protocol  Statement 4 is correct: Banihal Pass in Jammu and
at

was signed in 2016 which is legally binding in Kashmir is located in the Pir Panjal range. It is also
cm

nature. called the Jawahar tunnel. It connects the Kashmir


valley to the outer Himalayas.
ps
.u

PRAYAS TEST 9 31
w
w
w
Contact us : info@onlyias.com

OnlyIAS Nothing Else Visit : dpp.onlyias.in


Contact : +91-7007 931 912

Q.62) Ans: b Q.65) Ans: c


Exp: Exp:
 Statement 1 is incorrect: Bhabar is a narrow area  Statement 1 is incorrect: Vindhya and Kaimur
running parallel to the foothills of the Shivalik. It is range are the source of origin of several rivers.
located at the end of its slope and hence river Most of the tributaries of Yamuna originate from
deposits are associated with this area. The here. However, Banas river originates from the
streams or rivers usually disappear in this area. Aravallis and is an exception here.
Terai areas are found south of the Bhabar areas  Statement 2 is incorrect: Malda fault exists
and the streams or rivers emerge in these areas. between the Rajmahal Hills/Chotanagpur Plateau
Hence these areas are more suitable for the and the Meghalaya/Shillong Plateau. It is
cultivation of crops as compared to Bhabar. responsible for the detachment of the
 Statement 2 is correct: The alluvial plains are Northeastern Plateau from the Deccan Plateau.
subdivided into Khadar and Bhangar. Bhangar are The northeastern plateau consists of Garo, Khasi,
the older alluvium and are deposited away from Jaintia hills in Meghalaya and the Karbi Anglong
the flood plains of the rivers. Whereas Khadar are hills of Assam.
the new alluvium deposits which are subjected to
frequent flooding. Hence, they are more fertile Q.66) Ans: c
with finer silt deposition. Exp:
 Statement C is correct: Harishchandra range is
Q.63) Ans: d located in the state of
Exp: Maharashtra, Nallamala hills form a part of the
 Statement 1 is incorrect: The northern plains are Eastern Ghats in Andhra
characterized by Bhabar, Tarai and the alluvial Pradesh, Dandakaranya is associated with Odisha,
plains. The alluvial plains are further divided into Chhattisgarh, Telangana and Andhra
Khadar and Bhangar. Hence, Bhabar and Terai do Pradesh, Ramgarh hills are found in the state of
not form a part of the alluvial plains. Jharkhand, Malyagiri hills are located in the state
 Statement 2 is incorrect: Most of the of Odisha, and Garhjat range are the mountain
Brahmaputra plains lie in the state of Assam and ranges stretching across Odisha, Jharkhand, and
not Arunachal Pradesh. Chhattisgarh.

Q.64) Ans: b Q.67) Ans: b


Exp: Exp:
 Statement 1 is correct: Western Ghats are  Statement 1 is incorrect: The presence of a wood
continuous with higher elevation as compared to fossil park at Atkal and the marine deposits of
Eastern Ghats. The height of the western ghats Brahmsar, Jaisalmer indicates the region was
increases southwards with the highest submerged under the sea and contains marine
peak Anamudi (Kerala) located on the Annamalai deposits. The underlying rocks of the region
hills of the Western ghats. appear as an extension of the peninsular
 Statement 2 is incorrect: Eastern ghats are plateau.
e

discontinuous and lower in elevation as compared  Statement 2 is correct: The Northern part of the
in

to the western ghats. It is highly eroded and desert area is sloping towards Sindh while the
nl

eastwards flowing rivers like Mahanadi, Godavari, southern part has a slope towards Rann of
l.o

Krishna, Kaveri forms delta. Kachchh in Gujarat. Hence, the area shows
 Statement 3 is incorrect: Bundelkhand region opposing slopes.
ria

forms a part of the Central Highlands of the


e

Peninsular Plateau and not the Deccan Q.68) Ans: b


at

Plateau. The region is associated with the states Exp:


cm

of Uttar Pradesh and Madhya Pradesh.  Statement 1 is incorrect: Western coasts of India
are an example of submergent coasts. Natural
ps
.u

PRAYAS TEST 9 32
w
w
w
Contact us : info@onlyias.com

OnlyIAS Nothing Else Visit : dpp.onlyias.in


Contact : +91-7007 931 912

ports and harbours are formed along the  Statement 2 is correct: Minicoy Island is the
submergent coasts. Kandla, Mangalore, Cochin, southernmost Island of the Lakshadweep and is
Mormagao, JLN port, etc. are some of the separated from the rest of the Lakshadweep
examples of natural ports. through the 9-degree channel.
 Statement 2 is incorrect: Gujarat has the
maximum coastline among the mainland states. If Q.72) Ans: b
the Union territories are also considered, then Exp:
Andaman and Nicobar Islands have the maximum  Statement 1 is correct: Catchment area can be
coastline. defined as the area from where the river gets its
 Statement 3 is correct: Northern circars formed a water. Hence, it is located near the source of the
part of the former British territory in the Madras river.
presidency. They are found along the states of  Statement 2 is incorrect: Drainage basin of a river
Andhra Pradesh and Odisha. It is a region lying is the entire geographical area drained by a river
between Mahanadi and Krishna rivers. including its tributaries.
 Statement 3 is incorrect: Drainage basin of a river
Q.69) Ans: c is composed of a larger area while the watersheds
Exp: are the small areas. The catchment areas of small
 Statement 1 is incorrect: Some parts of rivulets or rills is regarded as the watershed.
Lakshadweep Island lie above the 12-degree
channel. Q.73) Ans: b
 Statement 2 is incorrect: Duncan passage in the Exp:
Andaman is situated between south Andaman  Statement B is correct:
and the Little Andaman. o River Sindh originates from the Malwa Plateau and is
 Statement 3 is correct: Agatti Islands of a tributary of Yamuna. It flows through the states of
Lakshadweep is situated to the north of the Uttar Pradesh and Madhya Pradesh.
capital Kavaratti. o River Son originates from the Amarkantak plateau,
 Statement 4 is incorrect: Saddle peak is the Madhya Pradesh. It is the tributary of Ganga and
highest peak of Andaman and is situated in the meets Ganga at Arrah, Bihar.
North Andaman. o River Ken is the tributary of Yamuna and flows
through the Bundelkhand region of Uttar Pradesh and
Q.70) Ans: d Madhya Pradesh.
Exp: o Mayurakshi river originates in the Trikut hills of
 Statement 1 is correct: The Andaman and Nicobar Jharkhand. Hence, from west to east, the correct
group of Island are an extension of Arakan Yoma order of the above rivers is Sindh, Ken, Son, and
range which passes through Myanmar and the Mayurakshi.
Indian states of Assam, Nagaland, Manipur, and
Mizoram. Q.74) Ans: c
 Statement 2 is correct: Narcondam Island is Exp:
situated in the North Andaman is an example of a  Statement 1 is correct: Ganga river is an example
e

dormant volcano. of the antecedent river. These rivers originate


in

 Statement 3 is correct: Mt. Thullier is the highest prior to the upliftment of the surrounding
nl

peak of the Nicobar group of Islands. It is located landmass and maintain their original course
l.o

in the Great Nicobar group of Island. cutting through the mountainous region.
 Statement 2 is correct: Consequent streams on
ria

Q.71) Ans: c the other hand flow as per the slope of the
e

Exp: adjoining surface or mountains from where they


at

 Statement 1 is correct: The largest island in originated.


cm

Lakshadweep is the Adrott Island. Minicoy island


is the second largest island of Lakshadweep. Q.75) Ans: d
ps
.u

PRAYAS TEST 9 33
w
w
w
Contact us : info@onlyias.com

OnlyIAS Nothing Else Visit : dpp.onlyias.in


Contact : +91-7007 931 912

Exp: Kerala and Tamil Nadu. It drains at the


 Statement D is correct: Lakshadweep sea.
o The origin of river Mahanadi is near Sihawa,  Statement 2 is correct: Vembanad lake is the
Chattisgarh. It flows through the states of Madhya longest lake in India and the largest lake of
Pradesh, Chhattisgarh, and Odisha. Kerala. Vembanad-Kol wetlands is the second
o The origin of river Kaveri is in Brahmagiri hills, near largest wetland site in India after the SundarbAns.
Kodagu, Karnataka. It flows through the states of It is a part of backwaters of Kerala, locally called
Karnataka, Kerala, and Tamil Nadu. as Kayals. This is an example of a lagoon. Lagoons
o The origin of river Tapi is in Betul, Madhya Pradesh are the shallow coastal water bodies separated
and flows through the states of Maharashtra, Madhya from the sea by a series barrier islands/coral reef,
Pradesh, and Gujarat. etc.
o River Krishna originates near Mahabaleshwar,
Maharashtra and passes through the states of Q.79) Ans: b
Maharashtra, Karnataka, and Andhra Pradesh. Exp:
 Statement 1 is incorrect: Himalayan rivers are
Q.76) Ans: a perennial, but their discharge is not constant
Exp: throughout the year. Ganga river experiences
 Statement 1 is correct: River Godavari is the maximum discharge during the months of
largest river in the Peninsular river system. It August/September which coincides with the
originates near Nashik, Maharashtra and flows Indian monsoons while it has minimum discharge
through the states of Maharashtra, Madhya during January to June months.
Pradesh, Chhattisgarh, Odisha, and Andhra  Statement 2 is correct: Harike wetlands are
Pradesh. The major tributaries of Godavari are located at the confluence of Satluj and Beas rivers.
Penganga, Manjira, Pranhita, and Indravati. It is located in Punjab and is a Ramsar site.
 Statement 2 is  Statement 3 is incorrect: Verinag spring at the
incorrect: River Mandovi/Mahadayi originates at foothills of Pir Panjal range near Kashmir valley is
the Bhimgad, Karnataka and flows through the the origin of Jhelum. Verinag spring is in Anantnag
states of Goa, Maharashtra, and Karnataka. It is district of Jammu and Kashmir.
regarded as the lifeline of Goa. It is a west flowing
river that drains into Arabian Sea and does not Q.80) Ans: d
form a delta. The famous Dudhsagar falls are Exp:
located on this river.  Statement 1 is incorrect: Renuka lake is the
largest natural lake situated in the state of
Q.77) Ans: c Himachal Pradesh. It is a Ramsar wetland site.
Exp:  Statement 2 is incorrect: Renuka wetlands are not
 Statement 1 is incorrect: Vamsadhara is an east the only wetland sites of Himachal Pradesh. There
flowing river that originates in Odisha. It flows are three such sites in Himachal Pradesh, namely,
through the states of Odisha and Andhra Pradesh. Pong Dam in Kangra, Renuka in Sirmaur,
 Statement 2 is incorrect: Pennar is an east flowing and Chandertal in Lahaul and Spiti.
e

river that originates in Nandi hills in Karnataka. It


in

flows through the state of Karnataka and Andhra Q.81) Ans: b


nl

Pradesh. Exp:
l.o

 Statement B is correct: Among the given


Q.78) Ans: b rivers, Rind, Sengar, and Sindh are the
ria

Exp: tributaries of Yamuna. Raidak river is a


 Statement 1 is incorrect: Bharathapuzha is the
e

tributary of Brahmaputra. Son river is the


at

largest river of the state of Kerala while Periyar is second largest tributary of Ganga after river
cm

the second largest. Bharathapuzha originates near Yamuna.


Annamalai hills and flows through the states of
ps
.u

PRAYAS TEST 9 34
w
w
w
Contact us : info@onlyias.com

OnlyIAS Nothing Else Visit : dpp.onlyias.in


Contact : +91-7007 931 912

Q.82) Ans: b important in tracing the evolution of the


Exp: Himalayan river system.
 Statement 1 is incorrect: Rivers originating in the  Statement 3 is incorrect: There exists a slight (not
Amarkantak Plateau are an example of the radial sharp) tilt in the Deccan Plateau from west to
drainage system. Radial drainage system is east, which is responsible for most of the rivers
formed when a river originating from a hill draining the Bay of Bengal.
discharges its water in all the  Statement 4 is incorrect: River Tapi originates in
directions. Centripetal drainage patterns are Betul, Madhya Pradesh and flows through the
formed when the rivers discharge their waters states of Maharashtra, Madhya Pradesh, and
from all the directions to a particular lake or a Gujarat. Nearly 79 percent of its drainage basin is
depression. in the state of Maharashtra.
 Statement 2 is correct: Narmada river is the
largest west flowing river of the peninsula India. It Q.85) Ans: c
rises at Amarkantak plataeu and flows through Exp:
Madhya Pradesh, Maharashtra and Gujarat. The
river flows through the rift valley between
Vindhya and Satpura hill ranges before falling into
the Gulf of Cambay in the Arabian Sea. It forms
the traditional boundary between North and
South India. Importanat tributaries - Shakkar ,
Hiran, Tawa, Kolar etc. Son river originates in
Amarkantak plateau (Madhya Pradesh). Its source
is close to the origin of the Narmada. It flows
northwest for a while and then turns sharply
eastward and after flowing parallel to kaimur  Statement 1 is not correct: Western disturbances
range it joins the Ganga river at Patna (Bihar). are weak temperate or Extra-Tropical (not
Important tributaries of Son - Johilla, Gopat, tropical) frontal cyclones that originate over the
Rihand, Kanhar and North Koel. Mediterranean sea, gain moisture from the
Caspian Sea and the Black Sea and cause winter
Q.83) Ans: b rainfall in the North Western part of India. The
Exp: Himalayas act as orographic barriers.
 Statement B is correct: River Penganga is a  Statement 2 is not correct: The subtropical
tributary of Godavari and has its origin in westerly jet stream drives this low pressure
Maharashtra. Penneru river originates from the (cyclones cannot develop in High pressure
Nandi hills of Karnataka. River Tungabhadra is a regions as these regions are characterized by
tributary of Krishna. Manjira is a tributary of subsidence of wind) system towards the east of
Godavari that originates near Balaghat in the Mediterranean sea across Afghanistan,
Ahmednagar, Maharashtra. The correct sequence Pakistan and North West India. These storms are
of these rivers in the north-south direction is residual frontal cyclones which move at the height
e

Penganga, Manjira, Tungabhadra, and Penneru. of 2000 metres from the mean sea level.
in

 Statement 3 is correct: Rabi crops during the


nl

Q.84) Ans: b winter season are greatly influenced by the


l.o

Exp: arrival of western disturbance. It


 Statement 1 is incorrect: Narmada and Tapi are brings cloudiness and precipitation in many parts
ria

the west flowing rivers that flow through the rift of north India during rabi season. Rabi crops,
e

valley between the Vindhya and the Satpura particularly wheat crop, down under rainfed
at

ranges. conditions is greatly benefited by the arrival of


cm

 Statement 2 is correct: Lacustrine (related to Western Disturbances.


lakes) are found from the Shivalik. They are
ps
.u

PRAYAS TEST 9 35
w
w
w
Contact us : info@onlyias.com

OnlyIAS Nothing Else Visit : dpp.onlyias.in


Contact : +91-7007 931 912

Q.86) Ans: d Q.87) Ans: a


Exp: Exp:
 Statement 1 is correct: Large scale features,  Statement 1 is correct:
which are conducive for occurrence of severe o Both the Bay of Bengal and Arabian Sea experience
thunderstorms associated with cloudburst, are cyclonic events, thanks to their proximity to the Indian
predictable two to three days in advance. Ocean, where cyclones are a common phenomenon.
However, the specific location and time of However, when the two are compared, the Bay of
cloudburst can be predicted in a few hours in Bengal sees approximately five times as many
advance, when the genesis of a thunderstorm cyclones as its Western counterpart. In
has already commenced. addition, cyclones in the Bay are stronger and
o To detect these sudden developments, a Doppler deadlier. And what's more, nearly 58% of cyclones
Weather Radar (DWR), a powerful tool for time and formed in the Bay of Bengal reach the coast as
location specific prediction of cloudburst, can be compared to only 25% of those formed in the
deployed a few hours in advance. Coupled with Arabian Sea.
satellite imagery this can prove to be useful inputs for  Statement 2 is correct:
extrapolation of cloudbursts anywhere in India. o Monsoon withdrawal in India officially begins in early
 Statements 2 and 3 are correct: A September. Usually, soon after the withdrawal
cloudburst can occur anytime and at any process begins, Cyclones begin to form in either areas
place which is affected by convective of Arabian Sea or Bay of Bengal.
weather systems. Surrounded by oceAns on  During the late summer months, the sea surface
three sides, peninsular India is a favoured temperatures reach their highest levels and provide
location for the genesis of convective weather tropical cyclones with the energy they need
systems. In a short span of time, if the right to develop into major storms. Wind shear can tear
combinations of atmospheric conditions disturbances apart before they strengthen, which
like instability, moisture content and gradually fades later in the season, reaching a
triggering mechanisms are minimum in mid to late September. The lack of shear
available, cloudbursts are possible. combines with the increase in air temperature, water
o It is however not necessary that cloudbursts temperature and building moisture in the tropics.
happen only in the mountains or high altitudes as the  In contrast, these factors do not align in an optimum
weather systems have compatibility and criticality in manner during the pre monsoon season (April-May)
space and time. and hence the number of cyclones in these months is
o This implies that the smaller scale weather structures considerably low.
like tempest, tornado, cloudburst, etc. have shorter  As a matter of fact, Cyclone Fani (2019, Odisha) is
life span whereas large scale systems like tropical only the second severe cyclone in the last 118
cyclones have a longer life span.” years to form in the Bay of Bengal (BOB) in the
 Statement 4 is incorrect: month of April and cross over to the Indian mainland.
The IMD classifies rainfall intensity as 20-30 mm Between 1891-2017, only 14 severe tropical cyclones
rain per hour as an intense spell; 30-50 mm per formed in April over BOB and only one storm crossed
hour as a very intense spell; 50-100 mm per hour the Indian mainland.”
e

as an extremely intense spell; and over 100 mm  Statement 3 is correct:


in

per hour as a CLOUDBURST. o One of the prime reasons why the Bay of Bengal is
nl

o These massive coagulated clouds with heavy water more prone to Cyclogenesis is that the Bay of Bengal
l.o

content hover over a very small location unlike storms also welcomes cyclones formed far away in the
that spread anywhere from five to 50 km range and Pacific Ocean. Cyclones usually weaken over a
ria

convert into a downpour. The dead weight of the landmass. But due to lack of any such land between
e

cloud is so massive and unbearable that it the Pacific Ocean and the Bay of Bengal, cyclonic
at

simply collapses under its own weight with winds easily move into the bay. Thus Not all cyclones
cm

such ferocity and accompanied originate in the Indian ocean.


by thunder and lightning.
ps
.u

PRAYAS TEST 9 36
w
w
w
Contact us : info@onlyias.com

OnlyIAS Nothing Else Visit : dpp.onlyias.in


Contact : +91-7007 931 912

Q.88) Ans: a degrees above the normal maximum temperature of


Exp: a location, it is declared as a heatwave.
 Statement 2 is incorrect: Heatwaves are common
over the Core Heatwave Zone (CHZ) — Rajasthan,
Punjab, Haryana, Chandigarh, Delhi, West Madhya
Pradesh, Uttar Pradesh, Chhattisgarh, Orissa,
Vidarbha in Maharashtra, parts of Gangetic West
Bengal, Coastal Andhra Pradesh and Telangana, as
categorised by India Meteorological Department.
o The regions in the extreme north, northeast and
southwestern India are less prone to
heatwaves. Heat waves are rare over the peninsular
Statement 1 is correct and 2 is incorrect: region because of Maritime conditions.
o An MJO is an oceanic-atmospheric phenomenon  Statement 3 is correct: Summer season reaches
which affects weather activities across the globe. its peak by May 15 in India, when the day
It brings major fluctuation in tropical weather on temperatures across north, west, and central
weekly to monthly timescales. India cross 40 degrees and hover close to 45
o The MJO can be defined as an eastward degrees then on. In 2020, north India did not
moving 'pulse' of clouds, rainfall, winds and pressure experience such temperatures till May 21.
near the equator that typically recurs every 30 to 60 o It was mainly because of the continuous inflow of
days.It’s a traversing phenomenon and is most Western Disturbances that influenced the weather in
prominent over the Indian and Pacific OceAns. the north till as late as April. There have
 Statement 3 is incorrect: been frequent passing of Western Disturbances over
o The Indian Ocean Dipole (IOD), El Nino and MJO are the north, appearing after every five to seven days.
all oceanic and atmospheric phenomena, which affect Originating in the Mediterranean Sea, Western
weather on a large scale. The El-Nino is known to Disturbances are eastward-moving winds that blow in
cause droughts in India. However if the periods of El lower atmospheric levels. They thus affect the local
Nino overlap with the convective phase of the weather of a region during its onward journey.
Madden Julian oscillations, the effect of El Nino can  Statement 4 is correct: In 2020, Cyclone Amphan,
be reduced to some extent especially in the Arabian which hit the coasts of Northern Odisha and West
Sea cost regions where it rains heavily when the MJO Bengal, had a huge role in triggering heat waves
passes over the Indian Ocean. But the el across North India. Cyclone Amphan, which was a
Nino’s impact is felt across India while the madden- massive SuperStorm covering 700 kms, managed
julian oscillation’s impact is limited to only some to drag maximum moisture from over the Bay of
parts of India for a period of 30 to 60 days. Bengal, entire South Peninsula, parts of Central
Therefore, a complete nullification is not possible. India and to some extent, even from the Arabian
Sea
Q.89) Ans: b o All the moisture that was otherwise built during the
Exp: thunderstorm and rainfall, got gradually depleted
e

 Statement 1 is correct: Heat Waves occur over from over vast areas as the storm advanced towards
in

India between March and June. Meteorologists West Bengal and Bangladesh. It thus triggered dry
nl

define a heatwave event in reference to the north-westerly winds to blow over Rajasthan, Punjab,
l.o

normal maximum (day) temperature for a Madhya Pradesh, Uttar Pradesh and Maharashtra
location. causing severe heatwave.
ria

o For example, Heatwaves are declared in the plains if


e

the temperature crosses 40 degrees Celsius. Over Q.90) Ans: c


at

the hills, the threshold temperature is 30 degrees Exp:


cm

Celsius. When the day temperature jumps by 4 to 5


ps
.u

PRAYAS TEST 9 37
w
w
w
Contact us : info@onlyias.com

OnlyIAS Nothing Else Visit : dpp.onlyias.in


Contact : +91-7007 931 912

SHOWERS of Kerala in the month of April


and May are examples of norwesters .
 Statement 3 is incorrect: The rainfall in these
storms is beneficial for pre-kharif crops grown in
india. The tea cultivated in Assam and
the jute and rice and tea cultivated in West
Bengal and Bangladesh benefit from these
showers. As a matter of fact, the reason they are
named Mango showers or cherry blossom
showers in the South Indian States is because
they help and their ripening.

Q.92) Ans: b
Exp:
 As can be seen in the map here, the Monsoon hits
the Andaman and Nicobar islands, a full week
before the Arabian Sea branch hits the Kerala
coast (mainland of India).

 Western Rajasthan is the last place (Mid July)


where the monsoon reaches - which partially
explains the arid conditions.

Q.91) Ans: a
Exp:
 Statement 1 is correct: Norwesters are isolated
rainfall and thunderstorm events which occur in
India and Bangladesh, often with violent  Option 1 is incorrect
hurricane-speed winds. Typically, such storms
originate in a low-pressure area during the peak o As can be seen in the figure here, the Bay of Bengal
summer season. The Low pressure intensifies Branch travels almost parallel to the eastern ghats
when the sun beats down on the region. and meets the Eastern Himalayas head on. From
Therefore, they generally occur just before sunset there on, it turns left and enters the Gangetic plain
or just a few hours after it, when thick dark black while making its way towards Punjab, Haryana and
clouds start appearing over the South Western even Jammu and Kashmir. Thus, there is no rainfall in
sky and then bring gale-speed wind with torrential Western Ghats, caused by the Bay of Bengal branch.
rain, often with hail, but spanning only a short  Option 2 and 3 are correct
period of time. o The Western Ghats face the full brunt of cyclones
 Statement 2 is incorrect: Kal Baisakhi originating in the Arabian Sea (Nisarga,2020) and the
e

low pressure depressions which intensify into deep


in

(Norwesters in the Gangetic Plains) originates


depressions.
nl

over Bihar and Jharkhand area, in the chota


nagpur plateau and moves eastwards and strikes  Interestingly, Cyclones Originating in the Bay of
l.o

West Bengal and Odisha. These are extremely Bengal can also cause rain in the Western Ghats. For
ria

severe in nature and take Chhattisgarh under its example cyclone Amphan in 2020, caused rains
purview as well. in Karnataka, Kerala and Tamil
e

o However, they are also common Nadu. Similarly cyclone Gaja, hit the cost of Tamil
at

in Kerala, Tamil Nadu and coastal Nadu in 2018 and wreaked havoc in the districts
cm

Karnataka. The Mango Showers or of coimbatore, Dindigul (Western ghats) etc.


the CHERRY BLOSSOM
ps
.u

PRAYAS TEST 9 38
w
w
w
Contact us : info@onlyias.com

OnlyIAS Nothing Else Visit : dpp.onlyias.in


Contact : +91-7007 931 912

 Similarly, Deep depressions in Bay of Bengal can


channel
cause rainfall in Kerala and Tamil Nadu in the Western
Ghats Region.
 Option 4 is incorrect:
o The western disturbances are generally limited to the  9 degree
 Minicoy and Lakshadweep
northern and North Western parts of the country. channel
Hence, Western ghats do not get rainfall from these
weak extra-tropical cyclones. At times however,
when western disturbances become more intense in  10 degree  Andaman and Nicobar
the Indian Region, they can extend even upto 15 channel Islands
degree north Latitude, resulting in rainfall in Western
Maharashtra ( (Western ghats) , Gujarat and the  Statement 3 is wrong: Saddle peak is the highest
entire Madhya Pradesh to the south. However, this is in North Andaman and Mt.Thuillier is the highest
not an annual occurrence and is an exception rather peak in the Great Nicobar Islands.
than a norm.  Statement 4 is wrong: Havelock island is the
 Option 5 is correct: largest island in the union territory. Recently it
was renamed as Swaraj Dweep.

Q.94) Ans: c
Exp:
 Pair 1 is Incorrect : Son river is a perennial river
located in central India originating in the
Amarkantak hills in Madhya Pradesh and finally
merges with the Ganga in Bihar. Tributaries of Son
river -Rihand River, Johila River, North koel
River, Gopad River, Kanhar river. South Koel
River, a tributary of the Brahmani River.
 Pair 2 is correct : The river Brahmaputra
originates in Himalayan Lake MAnsarovar in Tibet
to the outfall in the Bay of Bengal. Flowing
o The North Eastern monsoon winds pick up moisture southwest, it receives its main left bank
from the Bay of Bengal and have enough force to tributaries, viz., Dibang or Sikang and Lohit;
cause rain in the interior districts of Tamilnadu thereafter, it is known as the Brahmaputra.
(Nilgiris, Coimbatore, Theni) and even parts of Among the tributaries SubAnsiri, Manas, Jia
Kerala. Bharali, Pagladiya, Puthimari and Sankosh etc. are
snow fed.
Q.93) Ans: c  Pair 3 is correct : The principal tributaries joining
Exp: Krishna are the Ghataprabha, the Malaprabha, the
 Statement 1 is correct – Andaman and Nicobar Bhima, the Tungabhadra and Musi. Musi River is a
e
in

Islands were formed due to the collision between tributary of the Krishna River in the Deccan
Plateau flowing through Telangana state in
nl

Indian Plate and Burma Minor Plate. Therefore,


Andaman Islands were an extension of the India. Hyderabad stands on the banks of Musi
l.o

submerged Arakan Yoma mountains of Myanmar. river.


ria

 Statement 2 is wrong - Andaman and Nicobar  Pair 4 is Incorrect : The Mahanadi is one of the
Islands are separated by a 10 degree channel. major east flowing peninsular rivers draining into
e

the Bay of Bengal. The principal tributaries of the


at

Mahanadi river are; Sheonath, Jonk, Hasdeo,


cm

 8 degree  Minicoy and Maldives Mand, Ib, Ong and Tel. Kangsabati is a tributary
of Damodar river.
ps
.u

PRAYAS TEST 9 39
w
w
w
Contact us : info@onlyias.com

OnlyIAS Nothing Else Visit : dpp.onlyias.in


Contact : +91-7007 931 912

Q.95) Ans: a Bengal. River Rangeet originated in Sikkim and is


Exp: the main tributary of the Teesta.
 Statement 1 is Incorrect : The Chambal  Pair 4 is Incorrect : River Sarda or Saryu rises in
river originates at Janapav, south of Mhow town, the Milam glaciers in the Nepal Himalayas where
on the south slope of the Vindhya Range in it is known as Goriganga. Along the Indo-Nepal
Madhya Pradesh. Chambal flows north-northeast border, it is called kali or chauk, where it joins the
through Madhya Pradesh, running for a time Ghaghara.
through Rajasthan, then forming the boundary  Pair 5 is correct : Brahmaputra river originates in
between Rajasthan and Madhya Pradesh. the Chemayungdung Glacier of the Kailash range
 Statement 2 is Incorrect : The Chambal River near the MAnsarovar Lake. It is entered by the
moves into Uttar Pradesh and runs for around 32 names of Siang and Dihang in India. And after it is
km prior to meeting the Yamuna River in Etawah joined by its two main tributaries, the Dibang and
district of Uttar Pradesh at an altitude of 122 m, the Lohit, it is known by the name
creating a portion of the greater Gangetic of Brahmaputra. It flows in Bangladesh by the
drainage system. name of Jumna.
 Statement 3 is Incorrect : The Banas is a river
which lies entirely within the state of Rajasthan in Q.97) Ans: d
western India. It is a left bank tributary of the Exp:
Chambal River, itself a tributary of the Yamuna,  Statements 1 and 2 are correct : Amarkantak
which in turn merges into the Ganga. The Banas is Hills Radial Drainage Pattern - River Son, River
approximately 512 kilometres in length. Narmada, River Johilla and River
 Statement 4 is correct : Badland topography is a Mahanadi. The Amarkantak region is a unique
major feature of the Chambal valley is natural heritage area and is the meeting point of
characterized by an undulating floodplain, gullies the Vindhya and the Satpura Ranges, with the
and ravines.Ravines are a type of fluvial erosional Maikal Hills being the fulcrum. This is where the
feature and are formed as a result of constant Narmada River, the Son River and
vertical erosion by streams and rivers flowing over Johila River (Tributary of Son) emerge.
semi-arid and arid regions.  Statement 3 is correct : Rajkot Hills Upland
Radial Drainage Pattern - Machhu, Bhagawan,
Q.96) Ans: c Bhander, Shetrunji and River Mandav.
Exp:  Statement 4 is incorrect : Vamsadhara River is an
 Pair 1 is correct : The Ganges as Alaknanda rises east-flowing river which originates in Kalahandi
in the southern Himalayas on the Indian side of district of Odisha, flows in Odisha, along its
the Tibet border in the Satopanth Glacier and boundary with Andhra Pradesh and finally joins
Bhagirath Kharak Glaciers. The source of the the Bay of Bengal at Kalingapatnam, Andhra
Alaknanda lies in the ranges above the temple Pradesh. It is the main river of north-eastern
town of Badrinath. Andhra region and the Boddepalli Rajagopala Rao
 Pair 2 is correct : Ghaghara originates in the Project was constructed on it to meet the
glaciers of mapchachungo glaciers. After irrigation needs of the region.
e

collecting the waters of its tributaries- Tila, Seti  Statement 5 is incorrect : Luni river Originates in
in

and Beri, it comes out of the mountain, cutting a the Naga hills of Aravalli Ranges near Pushkar
nl

deep gorge at Shishapani. The river sarda or kali valley in Ajmer District . It is also known as
l.o

joins it in the plain before it finally meets the Sagarmati . After crossing Govindgarh,it joins the
Gnaga at Chhapra. Saraswati river which originates from Pushkar lake
ria

 Pair 3 is Incorrect : Teesta river originates and from then on it is referred to as the Luni
e

at Pahunri (Teesta Kangse) Glacier in eastern River. The Luni river is Not part of any Radial
at

Himalaya and flows through Indian state of Sikkim Drainage system.


cm

and West Bengal and finally joins Brahmaputra in


Bangladesh to further drain into Bay of Q.98) Ans: d
ps
.u

PRAYAS TEST 9 40
w
w
w
Contact us : info@onlyias.com

OnlyIAS Nothing Else Visit : dpp.onlyias.in


Contact : +91-7007 931 912

Exp:  Statement 1 is Incorrect : India receives rainfall


 Statement 1 is Incorrect : Tamil Nadu Coast during two seasons. About 75 per cent of the
(South-East) receives heavy rains accompanied by country’s annual rainfall is received from the
stormy winds in the winter sometimes causing Southwest monsoon between June and
great damages to the crops and disrupting the September. North- East Monsoon occurs during
meAns of trAnsport. Rainfall comes from October to December, and is confined to the
the North East Monsoon and the Bay of Bengal Southern peninsula. It is also called the winter
Branch of Indian Monsoon as it flows parallel to monsoon and it is important for Tamil Nadu,
the coast. Puducherry, coastal Andhra Pradesh etc.
 Statement 2 is Incorrect : North West region of  Statement 2 is Incorrect : North- East Monsoon
India including Punjab, Haryana, Jammu and Winds characterized pleasant seasons with low
Kashmir , Himachal Pradesh , Northern Rajasthan temperature and low humidity and clear skies.
and western Uttar Pradesh receive moderate to They do not give extreme rain to any part of India
heavy rainfall due to westerly depression or except Tamil Nadu Coast.On the Other
western disturbances coming from hand, Retreating South West monsoon winds
Mediterranean sea. It is beneficial for Early are characterized by Oppressive heat and
ripening of Rabi crops, especially Wheat. Himalaya humidity known as “October Heat”.
receives a considerable amount of snowfall in
winter. Extra Edge by Only IAS
 North East Monsoon winds blow from the North east
Q.99) Ans: c Direction from the Land to the sea.
Exp:  Retreating South West Monsoon winds blow from the
 Statement 1 is correct : Karnataka’s Mountain South West direction but are not strong enough to
topography is broader than blow right into the northern plains. They withdraw in
Maharashtra. Greater Mountain width makes stages which results in decreasing rains.
Rain clouds travel a longer distance and hence
more time for water to coalesce and precipitate
as rainfall. Compared to this, Narrower width of
Western Ghats in Maharashtra causes Rain
bearing winds to cross over to the leeward side
quickly before rainfall can occur.
 Statement 2 is Incorrect : Arabian sea branch of
Indian South west monsoon strikes the coast of
Mumbai. Moving along the Narmada and Tapi
valleys, these winds cause rainfall in extensive
areas of central India. The Chotanagpur Plateau
receives a miniscule 15 cm rainfall from Arabian
sea branch. Thereafter they enter Ganga plains
and mingle with the Bay of Bengal Branch.
e

 Statement 3 is correct : Tamil Nadu coast


in

situated parallel to the Bay of Bengal Branch of


nl

South-west monsoon. It lies in the rainshadow


l.o

area of the Arabian sea branch of the South west


Monsoon. Rainfall comes from North East
ria

Monsoon and Not Bay of Bengal Branch of Indian


e

Monsoon
at
cm

Q.100) Ans: d
Exp:
ps
.u

PRAYAS TEST 9 41
w
w
w

You might also like